Download as pdf or txt
Download as pdf or txt
You are on page 1of 47

IDMP TEST SERIES-2023

High Yielding PRELIMS TEST SERIES

Test-05
Polity
Topic Covered:
❖ Election Commission of India Comptroller and Auditor General (CAG)
❖ Attorney General of India Advocate General of the State and CAG
❖ Finance Commission, National Commission for Backward Classes
❖ National Commission for Scheduled Castes, National Commission for Scheduled Tribes
❖ Special officer for Linguistic Minorities, Union Public Service Commission State Public Service
Commission

IDMP-2023
Contact us :info@onlyias.com

OnlyIAS Nothing Else Visit :dpp.onlyias.in


Contact : +91-7007 931 912

Q.1) According to Dr. BR Ambedkar, which of the Q.5) Consider the following statement regarding
following posts are called the "Bulwark of the representation of Comptroller and Auditor General of
democracy system of government in India"? India (CAG) in Parliament:
1. Union Public Service Commission 1. A minister can represent CAG in the upper house
2. Supreme Court of India of the Parliament but not in the lower house of the
3. Election Commission of India Parliament.
4. Comptroller and Auditor General of India 2. No minister can be called upon to take any
5. Attorney General of India responsibility for any action done by CAG.
6. Parliament Which of the statements given above is/ are correct?
Select the correct answer using the codes given a) 1 Only
below: b) 2 Only
a) 1, 2, 3 and 4 only c) Both 1 and 2
b) 1, 2, 4 and 6 only d) None of the above.
c) 2, 3, 5 and 6 only
d) 1, 3, 4 and 5 only Q.6) Consider the following statements regarding
removal of (Comptroller and Auditor General of India)
Q.2) Consider the following statement regarding CAG:
comptroller and auditor general of India: 1. CAG can be removed by the president on the same
1. The CAG is appointed by the president of India. grounds and in the same manner as a judge of the
2. The CAG, before taking over his office, is not bound Supreme Court.
to make and subscribe before the president an 2. He can be removed on the basis of proven
oath or affirmation. misbehaviour or incapacity.
3. The CAG holds office for a period of five years or up Which of the statements given above is/ are correct?
to the age of 65 years, whichever is earlier. a) Only 1
Which of the statements given above is/are correct? b) Only 2
a) 1 Only c) Both 1 and 2
b) 2 Only d) None of the above.
c) 3 Only
d) 1 and 3 only Q.7) Consider the following pair regarding
Comptroller and Auditor General of India:
Q.3) Which of the following statements is correct Article Provision
regarding re-appointment of the Comptroller and 1. 148 - Appointment of Comptroller and
Auditor General of India? Auditor General of India
a) He is not eligible for reappointment to any 2. 149 - Duties and powers of the comptroller
government office under any circumstances. and auditor general of India
b) He can hold office under the Government of India 3. 150 - Audit reports
but not under any State Government. 4. 151 - Form of accounts of the Union and of
c) He is eligible for reappointment. the states.
d) None of the above. Which of the pairs given above is/are correct?
a) 1 and 2 only
Q.4) Consider the following statement regarding b) 2 and 3 only
Comptroller and Auditor General of India: c) 1 and 4 only
1. The administrative expenses of the office of the d) 1 and 3 only
CAG charged upon the Consolidated Fund of India.
2. His salary and other service conditions are Q.8) Which of the following receipts and expenditures
determined by the Parliament. are audited by CAG of India?
Which of the statements given above is/ are correct? 1. All bodies and authorities substantially financed
a) 1 Only from the Central or state revenues.
b) 2 Only 2. Government companies.
c) Both 1 and 2 3. Other corporations and bodies, when so required
d) None of the above. by related laws.

DPP 2023_DAY 34 1
Download More Test Series & eBooks in Hindi & English from our telegram channel- https://t.me/upsc_success_time1
Contact us :info@onlyias.com

OnlyIAS Nothing Else Visit :dpp.onlyias.in


Contact : +91-7007 931 912

Select the correct answer using the codes given a) 1 and 2 only
below. b) 1 and 3 only
a) 1 and 2 only c) None of the above.
b) 2 and 3 only d) 3 only
c) 1 only
d) All of the above. Q.13) Consider the following statements regarding
Electronic Voting Machine (EVM):
Q.9) Comptroller and Auditor General of India (CAG) 1. The machines have been developed by C-DAC Pune
is accountable to which of the following? and Bharat Electronics Corporation of India Limited
a) President of India (BEL).
b) Lower house of the Parliament 2. In 1989 the Parliament amended the
c) Parliament Representation of the People Act, 1950 to provide
d) Supreme Court provision for the use of electronic voting machines
in elections.
Q. 10) Consider the following statements regarding Which of the above statements is/are correct?
comptroller and Auditor General of India: a) 1 only
1. Departments are authorized to draw money by b) 2 only
issuing cheques without specific authority from the c) Both 1 and 2
CAG. d) None of the above.
2. The duties and powers of the CAG of India
regarding issuing money from consolidated funds Q.14) Which of the following provisions makes the
are equivalent to that of the CAG of Britain. Comptroller and Auditor General of India as an
Which of the statements given above is/are incorrect? independent body?
a) 1 only 1. He is provided with the security of tenure.
b) 2 only 2. He held his office till the pleasure of the president.
c) Both 1 and 2 3. His salary and other service conditions are
d) None of the above. determined by the Parliament.
4. Ministers can represent the CAG in Parliament.
Q.11) CAG has no role in the audit of which of the Select the correct answer using the code given below.
following corporations? a) 1 and 3 only
1. Life Insurance Corporation of India b) 1, 3 and 4 only
2. Reserve Bank of India c) 2 and 3 only
3. Food Corporation of India d) 1, 2, 3 and 4
4. Air India
5. Oil and Natural Gas Commission Q.15) With reference to the duties and power of
6. State Bank of India Comptroller and Auditor General of India, Consider
Select the correct answer using the codes given the following statements:
below: 1. He audits the accounts related to all expenditure
a) 1, 2, 3 and 6 only from the Consolidated Fund of India.
b) 2, 4 and 5 only 2. He audits the receipts and expenditure of the
c) 2, 3 and 5 only Centre and each state.
d) 1, 4, 5 and 6 only 3. He audits all expenditure from the Contingency
Fund of India and the Public Account of India.
Q.12) Consider the following statements regarding 4. He submits his audit reports relating to the
Model Code of Conduct: accounts of the Centre to the President.
1. Model code of conduct is enforceable by law. Which of the statements given above is/are correct?
2. Model code of conduct is Constitutional power to a) 1 and 2 only
the Election Commission for conducting fair b) 2, 3 and 4 only
elections. c) 1, 2, 3 and 4
3. The model code of conduct is legally binding under d) 1, 3 and 4 only
the Representation of People Act, 1951.
Which of the above statements is/are correct?

DPP 2023_DAY 34 2
Download More Test Series & eBooks in Hindi & English from our telegram channel- https://t.me/upsc_success_time1
Contact us :info@onlyias.com

OnlyIAS Nothing Else Visit :dpp.onlyias.in


Contact : +91-7007 931 912

Q.16) Which are the following is/are the conditions 2. The administrative expenses of the office of CAG is
laid by the Indian Constitution to be appointed as subjected to a vote of Parliament.
Advocate General of State? 3. The conditions of service of persons serving in the
1. He must be a citizen of India. Indian Audit and Accounts Department is
2. He must have held a judicial office for ten years or prescribed by CAG.
been an advocate of a high court for ten years. Which of the statements given above is/are incorrect?
3. He can be an eminent jurist, in the opinion of the a) 1 and 2 only
Governor. b) 1 and 3 only
Select the correct answer using the codes given c) 2 only
below. d) 1, 2 and 3
a) 1 and 2 only
b) 2 and 3 only Q.21) Which of the following are the parts of CAG's
c) 1 and 3 only Oath as provided by the third Schedule of the
d) 1,2 and 3 Constitution?
1. To uphold the Constitution and the laws
Q.17) This body is known as a friend, philosopher and 2. To uphold sovereignty and integrity of India
guide to the Public Accounts Committee of 3. To devote himself to the service and well-being of
Parliament. This body ascertains and certifies the net people
proceeds of any tax or duty. Which of the following Select the correct answer using the codes given
offices is described in the above statements? below.
a) Comptroller and Auditor General a) 1 and 2 only
b) Finance Commission b) 1 and 3 only
c) Department of Economic affairs c) 2 and 3 only
d) NITI aayog d) 1,2 and 3

Q.18) CAG enables in ensuring the financial Q.22) Consider the following statements:
accountability of the executive to the 1. The contempt of court proceeding against an
legislature. Considering such importance, the CAG individual cannot be initiated without the consent
office should be independent of the Executive. This of the Attorney General or Solicitor General before
independence and impeachment process of CAG is the Supreme Court, if moved by a private
substantially modelled upon which of the following individual.
elements? 2. The term Contempt of Court is nowhere defined in
a) Government of India Act,1935 our Indian Constitution.
b) American Constitution 3. The Supreme Court and High Courts can take suo-
c) Canadian Constitution moto cognizance of the Court of contempt.
d) Australian Constitution Which of the statements given above is/are correct?
a) 1 and 2 only
Q.19) Which of the following Constitutional posts b) 2 and 3 only
holds office during the pleasure of the President? c) 1 and 3 only
1. Comptroller and Auditor General (CAG) d) 1, 2 and 3
2. Attorney General of India
3. Advocate General of the State Q.23) Which of the following offices enjoys all the
Select the correct answer using the codes given privileges and immunities that are available to a
below. member of Parliament?
a) 1, 2 and 3 1. Comptroller and Auditor General of India
b) 1 and 2 only 2. Attorney General of India
c) 2 only 3. Solicitor General of India
d) 2 and 3 only Select the correct answer using the codes given
below.
Q.20) Consider the following statements: a) 1 and 2 only
1. CAG submits its audit reports to the Parliament. b) 2 only
c) 1, 2 and 3

DPP 2023_DAY 34 3
Download More Test Series & eBooks in Hindi & English from our telegram channel- https://t.me/upsc_success_time1
Contact us :info@onlyias.com

OnlyIAS Nothing Else Visit :dpp.onlyias.in


Contact : +91-7007 931 912

d) 2 and 3 only 3. He holds office during the pleasure of the


governor.
Q.24) In context of office of Attorney General of India, Which of the statements given above is/are correct?
Consider the following statements: a) 2 and 3 only
1. The Attorney General of India is considered as a b) 2 only
member of Cabinet, like in the case of the U.K c) 1 and 3 only
government. d) 1, 2 and 3
2. He should not advise any ministry or department,
unless the proposal or reference is received Q.28) With the reference to Advocate General of
through the department of Personnel and training. state, Consider the following statements:
Which of the statements given above is/are correct? 1. Advocate General of state must be a citizen of India
a) 1 only and must have held a judicial office for ten years.
b) 2 only 2. Advocate General of state may be removed by the
c) Both 1 and 2 governor at any time.
d) Neither 1 nor 2 3. The remuneration of the advocate general is not
fixed by the Constitution.
Q.25) With reference to the independence of CAG Which of the statements given above is/are correct?
office, consider the following statements: a) 1 and 2 only
1. He holds his office till the pleasure of the President. b) 2 and 3 only
2. He is not eligible for further office, either under the c) 1 and 3 only
Government of India or of any state after he has d) 1, 2 and 3
ceased to hold his office.
3. His salary and other service conditions are Q.29) With reference to the Advocate General of
determined by the President. state, consider the following statements:
Which of the statements given above is/are correct? 1. He gives advice to the government of the state
a) 3 only upon such legal matters.
b) 2 and 3only 2. He has the right to vote and speak and to take part
c) 2 only in the proceedings of both the Houses of the state
d) 1 and 3 only legislature.
3. He enjoys all the privileges and immunities that are
Q.26) With reference to the Comptroller and Auditor available to a member of the state legislature.
General of India (CAG), Consider the following Which of the statements given above is/are correct?
statements: a) 1 only
1. He is the guardian of the public purse and controls b) 2 and 3 only
the entire Money and financial system of Centre c) 1 and 3 only
and State. d) 1, 2 and 3
2. He upholds the Constitution of India and is one of
the bulwarks of the democratic system. Q.30) The Constitution of India envisages the Finance
Which of the statements given above is/are correct? commission as the balancing wheel of fiscal
a) 1 only federalism in India. In this context Consider the
b) 2 only following statements about it:
c) Both 1 and 2 1. The President of India establishes a finance
d) Neither 1 nor 2 commission only after the completion of the fifth
year.
Q.27) Consider the following statements regarding 2. They hold office for such a period as specified by
the appointment and term of Advocate General of the the president in his order.
State: 3. The President is empowered to determine the
1. He must be a person who is qualified to be qualifications of the members of the commission.
appointed a judge of a supreme court. Which of the statements given above is/are correct?
2. The term of office of the advocate general is not a) 1 and 2 only
fixed by the Constitution. b) 2 and 3 only
c) 2 only

DPP 2023_DAY 34 4
Download More Test Series & eBooks in Hindi & English from our telegram channel- https://t.me/upsc_success_time1
Contact us :info@onlyias.com

OnlyIAS Nothing Else Visit :dpp.onlyias.in


Contact : +91-7007 931 912

d) 1 and 3 only Q.34) Which of the following statements is correct


regarding the finance commission of India?
Q.31) In the context of Indian polity Consider the a) It is a Quasi-judicial and Constitutional body.
following statements: b) It is a Quasi-judicial and Statutory body.
1. A judge of the high court or one qualified to be c) It is a Judiciary and Constitutional body.
appointed as one. d) It is a Judiciary and Unconstitutional body.
2. A person who has specialized knowledge of finance
and accounts of the government. Q.35) With reference to the Finance Commission,
3. A person who has wide experience in financial Consider the following statements:
matters and in Administration. 1. It has one chairman and four other members.
4. A person who has special knowledge of economics. 2. Chairman is appointed by the president.
Which one of the above criteria or criterions is/are 3. They are eligible for reappointment.
required to be a Chairman of the Finance Which of the statements given above is/are correct?
Commission? a) 1 and 2 only
a) Any one of the above Criterion b) 2 and 3 only
b) Only 1 and 2 c) 1 and 3 only
c) All of the above d) 1, 2 and 3
d) None of the above
Q.36) Consider the following statement, with
Q.32) Consider the following statement about reference to the determination of the qualifications of
recommendations that the Finance Commission is the Chairman and members of the Finance
required to make. Commission:
1. It has the authority to recommend the distribution 1. There is no mention in the constitution regarding
of the net proceeds of taxes to be shared between the qualifications of the members of the
the Centre and the states. commission.
2. It has not been vested with the power to 2. The Constitution authorizes the Parliament to
recommend the principles that should govern the determine the qualifications of members of the
grants-in-aid to the states by the Centre. commission.
Which of the Statements given above is/are correct? 3. The Finance Commission Act 1951 has been passed
a) 1 Only by the Parliament to determine the qualifications
b) 2 Only of the members of the Commission.
c) 1 and 2 Both Which of the statements given above is/are correct?
d) None of the above a) 1 only
b) 2 only
Q.33) With reference to Finance Commission in India, c) 2 and 3 only
consider the following Statements d) All of the above.
1. Under Article 280 of the Constitution, it has been
endowed with Power to recommend net proceeds Q.37) Consider the following statement regarding
of devolution between center and states. qualification of Chairman of Finance Commission:
2. The recommendations of the Finance Commission 1. He should be a person having experience in public
are binding upon the central government affairs.
3. The recommendation of the Finance Commission 2. A judge of the high court or one qualified to be
of India is applicable only to the Central appointed as one.
government as there is a separate State Finance Which of the statements given above is/are correct?
Commission for each State. a) 1 only
Which of the statements given above is/are correct? b) 2 only
a) 1 and 2 only c) Both 1 and 2
b) 2 and 3 only d) None of the above.
c) 1 and 3 only
d) 1 only Q.38) The Finance commission submits its report to
whom of the following?
a) President of India

DPP 2023_DAY 34 5
Download More Test Series & eBooks in Hindi & English from our telegram channel- https://t.me/upsc_success_time1
Contact us :info@onlyias.com

OnlyIAS Nothing Else Visit :dpp.onlyias.in


Contact : +91-7007 931 912

b) Parliament Select the correct answer using the codes given


c) Ministry of Finance below:
d) Lower house of the Parliament a) 1, 2 and 3
b) 1 and 2 only
Q.39) Which of the following statements is NOT c) 2 and 3 only
correct regarding the Finance Commission of India? d) 1 and 3 only
a) Recommendation made by Finance Commission
advisory in nature for government. Q.43) Consider the following statement regarding
b) Finance commission balancing the wheel of fiscal 'National Commission for Backward Class':
federalism in India. 1. The NCBC would recommend only to the central
c) The Finance Commission Act 1951 has been passed govt. regarding inclusion or deletion of a particular
by the Parliament to determine the qualifications caste in the list.
of the members of the Commission. 2. The NCBC has the power of the civil court.
d) Member of the Finance Commission not eligible for Which of the statements given above is/are correct?
reappointment. a) 1 only
b) 2 only
Q.40) Which of the following is/are functions of the c) Both 1 and 2
Finance Commission? d) None of the above.
1. Suggestions on grants given to some States in lieu
of assignment of any share of the net proceeds in Q.44) Consider the following Constitutional
each year of export duty on jute and jute products. Amendment Acts (CAA):
2. Any other matter referred to it by the president in Constitutional Amendment Provision
the interests of sound finance. 1. 100th Constitutional Amendment Act - Land
3. Suggestions on Grants-in-aid to the states by the Boundary Agreement between India and
Centre from the consolidated fund of India. Bangladesh
Select the correct answer using the codes given 2. 101st Constitutional Amendment Act - Goods and
below: Service Tax
a) 1 and 2 only 3. 102nd Constitutional Amendment Act - National
b) 2 and 3 only Commission on Backward Class
c) 1 and 3 only 4. 103rd Constitutional Amendment Act -
d) All of the above Reservation For Economically Weaker Section
Which of the pairs given is/are correctly matched?
Q.41) Consider the following statement regarding a) 1, 2 and 3 only
'National Commission for Backward Class': b) 2 and 4 only
1. It is a five-member commission with one woman c) 1 and 3 only
member. d) All of the above.
2. All members of the commission are appointed by
the president. Q.45) Which of the following is/are duties and
3. The commission presents an annual report to the functions of the National Commission for Scheduled
President. Castes under the Article 338 of the Indian
Which of the statements given above is/are correct? constitution ?
a) 3 only 1. To inquire into specific complaints with respect to
b) 2 and 3 only the deprivation of rights and
c) 1, 2 and 3 2. To present to the parliament, annually and at
d) 1 and 2 only such other times as the Commission may deem fit,
reports upon the working of those safeguards.
Q.42) Which of the following is the basis of 3. To discharge such other functions in relation to the
backwardness of castes included in the Central List by protection, welfare and development and
the Commission? advancement of the Scheduled Castes.
1. Socially Select the correct answer using the codes given
2. Economically below.
3. Educationally a) 1 and 2 only

DPP 2023_DAY 34 6
Download More Test Series & eBooks in Hindi & English from our telegram channel- https://t.me/upsc_success_time1
Contact us :info@onlyias.com

OnlyIAS Nothing Else Visit :dpp.onlyias.in


Contact : +91-7007 931 912

b) 2 and 3 only b) 2 only


c) 1 and 3 only c) Both 1 and 2
d) 1, 2 and 3 d) Neither 1 nor 2

Q.46) While investigating the matters the National Q.50) Which of the following is/are functions of
Commission for Scheduled Caste has all the powers of National commission for Scheduled castes (SCs)?
a Civil Court trying a suit and in particular in respect 1. To investigate and monitor all matters relating to
of Which of the following matters? the constitution.
1. Summoning and enforcing the attendance of any 2. To participate and advise on the planning process
person from any part of India and examining him of socio-economic development of the SCs.
on oath. Select the correct answer using the code given below.
2. Requiring the discovery and production of any a) 1 only
documents. b) 2 only
3. Requisitioning any public record or copy thereof c) Both 1 and 2
from any court or office. d) Neither 1 nor 2
4. Issuing summons/communications for the ex
amination of witnesses and documents. Q.51) With the reference to the National Commission
Which of the statements given above is/are correct? for SCs, Consider the following statements:
a) 1, 2 and 3 only 1. The commission presents an biannual report to the
b) 2, 3 and 4 only president.
c) 1, 3 and 4 only 2. The President forwards any report of the
d) 1, 2, 3 and 4 Commission pertaining to the state government to
the state governor.
Q.47) With reference to scheduled Tribes in India Which of the statements given above is/are correct?
consider the following statements. a) 1 only
1. The Constitution defines the criteria for b) 2 only
recognition of Scheduled Tribes through the 89th c) Both 1 and 2
Constitutional amendment Act of 2003. d) Neither 1 nor 2
2. The largest number of tribal communities are
found in Punjab State. Q.52) The National Commission for Scheduled Castes,
Which of the above statements is / are not correct? while investigating any matter or inquiring into any
a) 1 only complaint, has all the powers of a civil court trying a
b) 2 only suit and in particular in respect of Which of the
c) Both 1 And 2 following matters?
d) Neither 1 Not 2 1. Summoning and enforcing the attendance of any
person from any part of India.
Q.48) The National Commission for Scheduled Caste 2. Requisitioning any public record from any court.
(NCSC) is required to present annual reports to - Select the correct answer using the code given below.
a) Parliament a) 1 only
b) President b) 2 only
c) Central Government c) Both 1 and 2
d) None of the above d) Neither 1 nor 2

Q.49) Consider the following statements regarding to Q.53) Consider the following statements about the
The National Commission for Scheduled Castes: National Commission for Scheduled Caste:
1. All the members including vice chairperson and 1. The President places all reports prepared by the
chairperson are appointed by the president by National Commission for scheduled Caste placed
warrant under his hand and seal. before the parliament.
2. Conditions of service and tenure of office are 2. The governor places it before the Chief minister,
determined by the Parliament. along with a memorandum explaining the action
Which of the statements given above is/are correct? taken on the recommendations of the
a) 1 only Commission.

DPP 2023_DAY 34 7
Download More Test Series & eBooks in Hindi & English from our telegram channel- https://t.me/upsc_success_time1
Contact us :info@onlyias.com

OnlyIAS Nothing Else Visit :dpp.onlyias.in


Contact : +91-7007 931 912

Which of the statements given above is/are correct? 1. It Measures to be taken over conferring ownership
a) 1 only rights in respect of minor forest produce to STs
b) 2 only living in forest areas.
c) Both 1 and 2 2. It can give licenses to the tribal groups to open bars
d) Neither 1 nor 2 and wine shops for their livelihood.
3. It is empowered to take measures to prevent
Q.54) With the reference to National Commission for alienation of tribal people from land.
Scheduled Caste, Consider the following statements: Select the correct answer using the code given below.
1. It is a six-member body consisting of a chairperson a) 1 and 3 only
and five other members. b) 3 only
2. It can issue summons for the examination of c) 1 and 2 only
witnesses and documents d) 1, 2 and 3
3. It can requisite any public record from any court or
office. Q.58) The National Commission for STs while
Which of the statements given above is/are correct? investigating any matter or inquiring into any
a) 1 and 2 only complaint, has all the powers of a civil court trying a
b) 2 and 3 only suit and in particular in respect Which of the following
c) 1, 2 and 3 matters?
d) 3 only 1. It issues summons for the examination of
witnesses and documents.
Q.55) Consider the following statements regarding 2. It summons and enforces the attendance of any
the National Commission for Scheduled Castes person from any part of India and examines him on
(NCSC): oath.
1. It seeks to provide safeguards against the Select the correct answer using the code given below.
exploitation of Scheduled Castes and Anglo-Indian a) 1 only
communities. b) 2 only
2. It specifies the inclusion or exclusion of castes in c) Both 1 and 2
the list of Scheduled Castes. d) Neither 1 nor 2
Which of the statements given above is/are incorrect?
a) 1 only Q.59) Consider the following statements regarding to
b) 2 only National Commission for STs:
c) Both 1 and 2 1. It is a five-member body consisting of a
d) Neither 1 nor 2 chairperson, vice-chairperson and three other
members.
Q.56) Consider the following statements: 2. Conditions of service and tenure of office are
1. National Commission for Scheduled Tribes is a determined by the Prime minister.
constitutional body. Which of the statements given above is/are correct?
2. Geographically and culturally, the STs are the same a) 1 only
as SCs but their problems are different from those b) 2 only
of SCs. c) Both 1 and 2
3. The separate National Commission for STs came d) Neither 1 nor 2
into existence in 2008.
Which of the statements given above is/are incorrect? Q.60) Consider the following statements with
a) 1 and 2 only reference to State Public service Commission:
b) 3 only 1. The Chairman and other members of the SPSC are
c) 2 and 3 only appointed by the Governor of the State.
d) 1, 2 and 3 2. The Chairman or any other member of SPSC shall
only be removed from his/her office by order of
Q.57) Which of the following functions of the National the Governor of the state.
Commission for STs is/are correct? Which of the statements given above is/are correct?
a) 1 only
b) 2 only

DPP 2023_DAY 34 8
Download More Test Series & eBooks in Hindi & English from our telegram channel- https://t.me/upsc_success_time1
Contact us :info@onlyias.com

OnlyIAS Nothing Else Visit :dpp.onlyias.in


Contact : +91-7007 931 912

c) Both 1 and 2 Q.64) Consider the following statements with


d) None of the above reference to Union Public service Commission:
1. All the members of the commission are appointed
Q.61) Consider the following statements with by the President of India.
reference to the Joint State public service Commission 2. The number of members of the commission and
(JSPSC) as mentioned under Article 315 of the Indian their terms and conditions of their service is
Constitution: determined by two houses of Parliament and
1. The resolution for setting up a Joint State public recommended to the President.
service Commission can only be introduced in Which of the statements given above is/are
Rajya Sabha i.e the Upper house of Parliament. Incorrect?
2. The Chairman and other members of JSPSC shall be a) 1 only
appointed by the President of India. b) 2 only
3. The JSPSC has to present the annual report to the c) Both 1 and 2
Governor of each of the States which are part of d) None of the above
the JSPSC.
Which of the statements given above is/are correct? Q.65) Consider the statements with reference to
a) 1 and 2 only Independent Functioning of Union Public Service
b) 2 and 3 only Commission (UPSC):
c) 1 and 3 only 1. The chairman and members enjoy the security of
d) 1,2 and 3 tenure.
2. The entire expenses of UPSC are charged on the
Q.62) With reference to the Union Public service Consolidated Fund of India.
Commission, consider the following statements: 3. The chairman of the UPSC on ceasing to hold office
1. The UPSC is consulted in making appointments to is not eligible for further employment in the
civil services and posts and in promotions and Government of India or any state.
transfers from one service to another depending Which of the statements given above is/are correct?
upon the suitability of candidates. a) 1 and 2 Only
2. It is Consulted on all disciplinary matters affecting b) 2 and 3 only
a person serving under the Government of India or c) 3 only
the Government of a State. d) 1,2 and 3
3. It is consulted while making reservations of
appointments or posts in favour of any backward Q.66) Consider the following statements with respect
class of citizens. to Union Public Service Commission (UPSC):
Which of the Statements given above is/are correct? 1. UPSC is an Advisory Body in terms of its
a) 1 and 2 only Recommendations.
b) 1 and 3 only 2. The reasons for non-acceptance of advice of UPSC
c) 3 only have to be laid before both the Houses in
d) 1, 2 and 3 only Parliament.
Which of the statements given above is/are correct?
Q. 63) Consider the following statements: a) 1 only
1. The government of India Act 1935 established b) 2 only
Public Service Commission’s separately for both c) Both 1 and 2
the central and the state government services. d) None of the above
2. Lee Commission in 1924 suggested for the 1st time
the establishment of an independent and impartial Q.67) Consider the matters on which advice of Union
Public Service Commission for India. Public service Commission can be sought by the
Which of the statements given above is/are correct? President of India:
a) 1 only 1. On all matters regarding the discipline and
b) 2 only punctuality of the employees of All India Services.
c) Both 1 and 2 2. The evaluation of the standard and efficiencies of
d) None of the above the candidates for appointment, promotion
or transfer on all civil posts.

DPP 2023_DAY 34 9
Download More Test Series & eBooks in Hindi & English from our telegram channel- https://t.me/upsc_success_time1
Contact us :info@onlyias.com

OnlyIAS Nothing Else Visit :dpp.onlyias.in


Contact : +91-7007 931 912

Which of the statements given above is/are correct? d) Jurisdiction of UPSC can be extended by an act
a) 1 only made by the parliament.
b) 2 only
c) Both 1 and 2 Q.71) It is situated in the northern part of Odisha’s
d) None of the Above Mayurbhanj district. Geographically, it lies in the
eastern end of the eastern ghat. The park derives its
Q.68) With reference to the Joint State Public Service name from ‘Semul’ which are red silk cotton trees
Commission (JSPSC) in India, consider the following growing in the area. The park has a high biodiversity
statements: with about 94 species of orchids and 3,000 species of
1. The establishment of JSPSC in India was first plants. Among them, Sal is a dominant tree species in
provided by the Government of India Act, 1919. the park, this is the best description of -
2. No such Joint State Public Service Commission has a) Manas National Park
yet been formed in India since independence b) Jim Corbett National Park
3. The JSPSC submits its report to the President who c) Periyar National Park
then sends it to respective Governors. d) Similipal Biosphere Reserve
4. A JSPSC can be created only by an Act of Parliament
and not by state legislatures. Q.72) Consider the following statements regarding to
Which of the above statements is/are correct? Green Bonds:
a) 1 and 2 only 1. Governments, corporations, financial institutions
b) 4 only and private individuals also can issue green bonds.
c) 3 and 4 only 2. Green bonds aim to put energy transition and clean
d) 1, 2 and 3 only energy at the heart of India’s economy.
3. Green bonds are designated bonds intended to
Q.69) With reference to UPSC consider the following encourage sustainability and to support climate-
statements: related projects.
1. The constitution has specified the strength or Which of the statements given above is/are correct?
composition of the UPSC. a) 2 only
2. One-half of the members of the Commission b) 1 and 2 only
should be such persons who have held office for at c) 2 and 3 only
least ten years either under the Government of d) 1, 2 and 3
India or under the government of a state.
3. Chairman and members of the commission hold Q.73) With the reference to the Society for worldwide
office for a term of five years or until they attain interbank financial telecommunication (SWIFT),
the age of 65 years, whichever is earlier. consider the following statements:
Which of the statements given above is/are correct? 1. SWIFT is a messaging network used by banks and
a) 1 only financial institutions globally.
b) 1 and 2 2. SWIFT assigns each member institution a unique
c) 2 only twelve-digit SWIFT ID code.
d) 2 and 3 3. SWIFT is merely a platform that sends messages
and does not hold any securities or money.
Q.70) Which of the following statements is not Which of the statements given above is/are correct?
correct? a) 1 and 2 only
a) UPSC serves all or any needs of a state on the b) 1, 2 and 3
request of the state governor and with the c) 2 and 3 only
approval of the president of India. d) 1 and 3 only
b) The chairman or member of UPSC is not eligible for
reappointment to that office after having Q.74) Which of the following statements is/ are
completed his first term. correct regarding Pradhan Mantri Kisan Man dhan
c) The chairman of the UPSC (on ceasing to hold Yojana?
office) is eligible for further employment in the 1. It is a voluntary and contributory pension scheme
government of India or state. for the Small & Marginal Farmers (SMFs).

DPP 2023_DAY 34 10
Download More Test Series & eBooks in Hindi & English from our telegram channel- https://t.me/upsc_success_time1
Contact us :info@onlyias.com

OnlyIAS Nothing Else Visit :dpp.onlyias.in


Contact : +91-7007 931 912

2. Small and Marginal Farmers must have cultivable 6. Address by the parliament presented.
land holding up to 2 hectares and the entry age of Select the correct answer from the options given
the scheme is 21 to 40 years. below.
3. No beneficiary has yet attained the age of 60 to be a) 1, 3, 2, 5, 4, 6
eligible for payment. b) 2, 1, 4, 3, 6, 5
Select the correct answer using the codes given c) 1, 2, 3, 4, 5, 6
below. d) 2, 4, 1, 6, 4, 5
a) 1 and 2 only
b) 2 and 3 only Q.79) Consider the following statements regarding to
c) 1 and 3 only Surety Bonds:
d) 1, 2 and 3 1. A surety bond is a legally binding contract entered
into by two parties: the principal, the obligee.
Q.75) Consider the following statements: 2. Surety bond is provided by the insurance company
1. Pradhan Mantri Urja Ganga Project aims to clean on behalf of the contractor to the entity which is
the Ganga River and Provide funds for the awarding the project.
Hydropower project. Which of the statements given above is/are correct?
2. Line of Credit (LOC) is a credit facility extended by a) 1 only
a bank or any other financial institution to a b) 2 only
government, business or individual customer. c) Both 1 and 2 only
Which of the statements given above is/are correct? d) Neither 1 nor 2
a) 1 only
b) 2 only Q.80) Recently Param Pravega term was seen in news,
c) Both 1 and 2 refers to -
d) Neither 1 nor 2 a) Anti - ballistic missile
b) Supercomputer
Q.76) National Urban Digital Mission, recently was in c) Drone
news, is launched by – d) Satellite launcher
a) Ministry of electronics and information
technology. Q.81) With the reference to the Neutrino, consider
b) Ministry of housing and urban affairs. the following statements:
c) Both A and B 1. A neutrino is a subatomic particle with negative
d) None of the above electric charge.
2. They do not interact with any matter and are not
Q.77) Authorised Economic Operator’ is a programme easy to catch.
of – Which of the statements given above is/are correct?
a) United Nations Conference of Trade and a) 1 only
Development b) 2 only
b) World Customs Organisation c) Both 1 and 2
c) World Trade Organisation d) Neither 1 nor 2
d) Organisation for Economic Cooperation and
Development Q.82) In the context of Stem cell, Consider the
following statements:
Q.78) Identify the correct chronology of the order 1. Stem cells are special human cells that are able to
under which a judge of a High Court is removed. develop into many different cell types.
1. Admission of the proposal by the 2. They can divide over and over again to produce
Speaker/Chairman. new cells.
2. Signing of the removal motion by the 3. Through Stem cell Human deficiency syndrome
parliamentarians. (HIV) can be cured permanently.
3. Passing of the motion for removal by both the Which of the statements given above is/are correct?
houses of the Parliament. a) 2 and 3 only
4. Setting up of a committee for investigation. b) 1, 2 and 3
5. President passing an order of removal. c) 1 and 3 only

DPP 2023_DAY 34 11
Download More Test Series & eBooks in Hindi & English from our telegram channel- https://t.me/upsc_success_time1
Contact us :info@onlyias.com

OnlyIAS Nothing Else Visit :dpp.onlyias.in


Contact : +91-7007 931 912

d) 1 and 2 only 3. The Supreme Court is not bound by its previous


judgements.
Q.83) Consider the following statements: Which of the statements given above is/are correct ?
1. The Indus Waters Treaty, 1960 is a water a) 1 and 2 only
distribution treaty between India and Pakistan b) 2 only
and brokered by the World Bank. c) 1 and 3 only
2. Under the Indus treaty, all the waters of three d) 1, 2 and 3.
eastern rivers namely Ravi, Sutlej, and Beas were
allocated to Pakistan for exclusive use. Q.87) Consider the following statements:
Which of the statements given above is/are correct? 1. Nai Roshni Scheme is the Central Sector Scheme
a) 1 only Started in 2012-13 and comes under the Ministry
b) 2 only of Minority Affairs.
c) Both 1 and 2 2. National Overseas Scholarship Scheme is the
d) Neither 1 nor 2 Central Sector Scheme which was launched in
2008.
Q.84) With reference to the Ayushman Bharat Digital Which of the statements given above is/are correct?
Mission, Consider the following statements: a) 1 only
1. The scheme will come under the Ayushman Bharat b) 2 only
Pradhan Mantri Jan Arogya Yojana. c) Both 1 and 2
2. Under the Mission, Indians as well as foreigners d) Neither 1 nor 2
will get a Health ID card that will store all medical
details of the person. Q.88) With the reference to the Lokayuktas, consider
3. Health ID is a randomly generated 14-digit number the following statements:
used for the purposes of uniquely identifying 1. It investigates allegations of corruption and mal-
persons. administration against public servants and is
Which of the statements given above is/are correct? 2. tasked with speedy redressal of public grievances.
a) 1 and 3 only 3. The Lokayukta is usually a former High Court Chief
b) 2 and 3 only Justice or former Supreme Court judge and has a
c) 1 and 2 only fixed tenure.
d) 1 , 2 and 3 4. Lokayukta cannot be dismissed nor transferred by
the government, and can only be removed by
Q.85) He was the founder of the Hindustan passing an impeachment motion by the
Republican Association also known as Hindustan Parliament.
Socialist Republican Association. He was a mentor for Which of the statements given above is/are correct?
revolutionaries like Chandrasekhar Azad and Bhagat a) 1 and 2 only
Singh. He along with Rashbehari Bose attacked b) 2 and 3 only
Viceroy Hardinge and injured him while he was c) 3 only
entering new capital of Delhi, this is best description d) 1, 2 and 3
of -
a) Sachindra Nath Sanyal Q.89) Which of the following Fundamental Duties
b) Ram Prasad Bismil is/are inculcated in the Indian constitution?
c) Bhagat Singh 1. To protect and improve the natural environment
d) Chandra shekhar Azad including forests, lakes, rivers and wildlife and to
have compassion for living creatures.
Q.86) Consider the following statements with respect 2. To develop the scientific temper, humanism and
to the powers of the Supreme Court. the spirit of inquiry and reform.
1. The Supreme Court can inquire into the conduct 3. To safeguard public property and to abjure
and behavior of the chairman and members of the violence.
Union Public Service Commission. Select the correct answer using the code given below.
2. The Supreme Court has the power to withdraw a) 1 only
cases pending before the High Court. b) 2 and 3 only
c) 1 and 3 only

DPP 2023_DAY 34 12
Download More Test Series & eBooks in Hindi & English from our telegram channel- https://t.me/upsc_success_time1
Contact us :info@onlyias.com

OnlyIAS Nothing Else Visit :dpp.onlyias.in


Contact : +91-7007 931 912

d) 1, 2 and 3 Which of the statements given above is/are correct?


a) 1 only
Q.90) With reference to Prime Minister’s b) 2 only
Development Initiative for North East (PM-DevINE), c) Both 1 and 2
new scheme launched in Union Budget 2022-23, d) Neither 1 nor 2
Consider the following statements:
1. Objective of the scheme is to enable livelihood Q.94) Consider the following statements:
activities for youth and women. 1. The Election Commission of India is a five-member
2. Under this scheme Construction of Aizawl bypass is body including chairperson and four other
one of the projects to be implemented. members.
Which of the statements given above is/are correct? 2. The Election Commission resolves the disputes
a) 1 only relating to splits/mergers of recognized political
b) 2 only parties and decisions cannot be challenged in the
c) Both 1 and 2 court of law.
d) Neither 1 nor 2 Which of the statements given above is/are incorrect?
a) 1 and 2 only
Q.91) Which of the following can be applied to b) 2 only
Synthetic Biology? c) 1 and 3 only
1. Developing synthetic organisms d) 3 only
2. Creating natural products in a lab
3. Nitrogen fixer for plants Q.95) Under which of the following conditions a
4. Car T cell therapy for cancer treatment person shall be disqualified for being elected as a
Select the correct answer using codes given below. Member of Parliament?
a) 1 and 3 only 1. If he holds any office of profit under the state
b) 2, 3 and 4 only government.
c) 1, 2, 3 and 4 2. If he has been convicted for an offence resulting in
d) 2 and 4 only imprisonment of 18 or more months.
Which of the statements given above is/are correct?
Q.92) In the context of cultural history of India, which a) 1 only
of the following statements is/are correct regarding b) 2 only
Hoysala Architecture style? c) Both 1 and 2
1. Hoysala architecture is a building style that evolved d) Neither 1 nor 2
between the 11th to 14th centuries under the
Central Hoysala Empire. Q.96) Which of the following are Fundamental Duties
2. Both open and closed mandapas can be found in of citizens mentioned in the constitution of India?
Hoysala architecture. 1. To develop scientific temper, humanism and the
3. Hoysala temples have circular pillars in their spirit of inquiry and reform.
mandapas. Each pillar has four sculpted figures on 2. To sing the National Anthem during morning
the top brackets. assembly in the school.
Select the correct answer using the codes given 3. To value and preserve the rich heritage of the
below. country's composite culture.
a) 1 and 2 only Which of the statements given above is/are correct?
b) 2 and 3 only a) 1 and 3 only
c) 1, 2 and 3 b) 1 only
d) 1 and 3 only c) 2 and 3 only
d) 1, 2 and 3
Q.93) Consider the following statements:
1. Poliomyelitis is a highly infectious viral disease that Q.97) Which of the following is/are necessarily the
largely affects children under 5 years. consequences of the proclamation of the president's
2. 2.Lassa fever is a zoonotic disease caused by the rule in a State?
Lassa bacteria and first discovered in 1969 in 1. Dismissal of the council of ministers in the state.
Nigeria. 2. Dissolution of the state Legislative Assembly.

DPP 2023_DAY 34 13
Download More Test Series & eBooks in Hindi & English from our telegram channel- https://t.me/upsc_success_time1
Contact us :info@onlyias.com

OnlyIAS Nothing Else Visit :dpp.onlyias.in


Contact : +91-7007 931 912

3. The president can assume to himself the powers


vested in the state high court.
Select the correct answer using the code given below.
a) 1 only
b) 1 and 3 only
c) 2 only
d) 1, 2 and 3

Q.98) Which of the following statements with


reference to high courts in India is/are correct?
1. The judgement and orders of a high court can only
be reviewed by the supreme Court of India.
2. As a court of record the judgements proceedings
and acts of a high court are admitted to be
evidentiary in nature.
3. A high court has the power to punish for contempt
of court.
Select the correct answer using the code given below.
a) 1 and 3 only
b) 2 only
c) 2 and 3 only
d) 1 only

Q.99) Consider the following statements about the


distribution of minerals in India:
1. Kerala has deposits of monazite and thorium.
2. Rajasthan is rich in building stones.
3. Assam valley is rich in Mineral oil reserves.
Which of the statements given above is/are correct?
a) 1 and 2 only
b) 2 and 3 only
c) 1 and 3 only
d) 1, 2 and 3

Q.100) Which of the following submarines are part of


project 75?
1. Kaveri
2. Vela
3. Khanderi
4. Vagir
Select the correct answer using codes given below:
a) 1 and 3 only
b) 1, 3 and 4 only
c) 2, 3 and 4 only
d) 1, 2, 3 and 4

DPP 2023_DAY 34 14
Download More Test Series & eBooks in Hindi & English from our telegram channel- https://t.me/upsc_success_time1
Download more eBooks & Test Series in Hindi and English
from our
Join Telegram- Telegram Channel-
https://t.me/upsc_success_time1
https://t.me/upsc_success_time1

Join Telegram Channel - Click Here


Contact us :info@onlyias.com

OnlyIAS Nothing Else Visit :dpp.onlyias.in


Contact : +91-7007 931 912

Q.1) Ans: a Independence of CAG:


Exp: The Constitution has made some provisions to
• According to Dr BR Ambedkar there are four main safeguard and ensure the independence of CAG:
posts which are called bulwark of Democratic • He is provided with the security of tenure.
system of government in India- • He can be removed by the president only in
o Union Public Service Commission accordance with the procedure mentioned in the
o Supreme Court of India Constitution.
o Election Commission of India • Thus, he does not hold his office till the pleasure of
o Comptroller Auditor General of India the president, though he is appointed by him.
• The Constitution of India through Article 148 • He is not eligible for further office, either under
provides for an independent office of the the Government of India or of any state, after he
Comptroller and Auditor General of India (CAG). ceases to hold his office.
• CAG is the head of the Indian Audit and Accounts • His salary and other service conditions are
Department. He is the guardian of the public determined by the Parliament. His salary is equal
purse and controls the entire financial system of to that of a judge of the Supreme Court.
the country at both the levels—the Center and the • Neither his salary nor his rights in respect of leave
state. of absence, pension or age of retirement can be
• His duty is to uphold the Constitution of India and altered to his disadvantage after his appointment.
laws of Parliament in the field of financial
administration. This is the reason why Dr B R Q.4) Ans: c
Ambedkar said that the CAG shall be the most Exp:
important Officer under the Constitution of India • Statement 1 is correct: The administrative
and he is one of the bulwark of the democratic expenses of the office of the CAG, including all
system of government in India. salaries, allowances and pensions of persons
serving in that office are charged upon the
Q.2) Ans: a Consolidated Fund of India. Thus, they are not
Exp: subject to the vote of Parliament.
• Statement 1 is correct: The CAG is appointed by • Statement 2 is correct: His salary and other service
the president of India by a warrant under his hand conditions are determined by the Parliament. His
and seal. He can resign any time from his office by salary is equal to that of a judge of the Supreme
addressing the resignation letter to the president. Court. Neither his salary nor his rights in respect of
He can be removed by the president on the basis leave of absence, pension or age of retirement can
of a resolution passed to that effect by both the be altered to his disadvantage after his
Houses of Parliament with special majority, either appointment. The conditions of service of persons
on the ground of proved misbehavior or serving in the Indian Audit and Accounts
incapacity. Department and the administrative powers of the
• Statement 2 is incorrect: The CAG, before taking CAG are prescribed by the president after
over his office, makes and subscribed before the consultation with the CAG.
president an oath or affirmation:
1. to bear true faith and allegiance to the Constitution Q.5) Ans: b
of India; Exp:
2. to uphold the sovereignty and integrity of India; • Statement 1 is correct: Further, no minister can
3. to duly and faithfully and to the best of his ability, represent the CAG in Parliament (both Houses).
knowledge and judgment perform the duties of his Because:
office without fear or favor, affection or ill-will; o The Constitution of India (Article 148) provides
4. to uphold the Constitution and the laws. for an independent office of the Comptroller and
• Statement 3 is incorrect: He holds office for a Auditor General of India (CAG).
period of six years or upto the age of 65 years, o He is the head of the Indian Audit and Accounts
whichever is earlier. Department.
o He is the guardian of the public purse and
Q.3) Ans: a controls the entire financial system of the
Exp:

DPP 2023_DAY 34 15
Download More Test Series & eBooks in Hindi & English from our telegram channel- https://t.me/upsc_success_time1
Contact us :info@onlyias.com

OnlyIAS Nothing Else Visit :dpp.onlyias.in


Contact : +91-7007 931 912

country at both the levels—the Center and the • Pair 3 is incorrect: The provision of form of
state. accounts of the Union and the states has been
o His duty is to uphold the Constitution of India made in Article 150 of the Constitution. According
and laws of Parliament in the field of financial to the article the accounts of the Union and the
administration. states shall be kept in such form as the President
• Statement 2 is correct: No minister can represent may on the advice of the Comptroller and Auditor
CAG in Parliament as well as can not be called General of India.
upon to take any responsibility for any action • Pair 4 is incorrect: 151- Audit reports:
done by him. o 151(1) - The reports of the Comptroller and
Auditor General of India relating to the accounts
Q.6) Ans: c of the union shall be submitted to the President
Exp: who shall cause them to be laid before each
• Statement 1 and 2 is correct: CAG can be removed house of parliament.
by the president on the same grounds and in the o 151(2) - The reports of the computer roller and
same manner as a judge of the Supreme Court. Auditor General of India relating to the accounts
• In other words, he can be removed by the of the state shall be submitted to the governor
president on the basis of a resolution passed to of the state who shall cause them to be laid
that effect by both the Houses of Parliament with before the Legislature of the state.
special majority, either on the ground of proven
misbehavior or incapacity. Q.8) Ans: d
• He holds office for a period of six years or up to the Exp:
age of 65 years, whichever is earlier. About CAG
• He can resign any time from his office by • CAG is the head of the Indian Audit and Accounts
addressing the resignation letter to the president. Department. He is the guardian of the public
purse and controls the entire financial system of
Q.7) Ans: a the country at both the levels—the Center and the
Exp: state.
• Pair 1 is correct: Article 148 of the Constitution • His duty is to uphold the Constitution of India and
provides for the independent office of the laws of Parliament in the field of financial
Comptroller and Auditor General of India. administration.
o 148(1) - appointment of CAG by president of • The Constitution (Article 149) authorizes the
India. Parliament to prescribe the duties and powers of
o 148(2) - oath and affirmation for CAG. the CAG in relation to the accounts of the Union
o 148(3) - salary and condition of service for the and of the states and of any other authority or
CAG. body.
o 148(4) - Non-eligibility of CAG for • Accordingly, the Parliament enacted the CAG’s
reappointment. (Duties, Powers and Conditions of Service) act,
o 148(5) - condition of service of personal serving 1971.
in the Indian Audit and accounts department. • So, CAG has the power to audits the receipts and
o 148(6) - all administrative expenses including all expenditure of the following:
salaries, allowances and pensions shall be o All bodies and authorities substantially financed
charged upon the consolidated fund of India. from the Central or state revenues;
• Pair 2 is correct: The Constitution (Article 149) o Government companies; and
authorizes the Parliament to prescribe the duties o Other corporations and bodies, when so
and powers of the CAG in relation to the accounts required by related laws.
of the Union and of the states and of any other
authority or body. Accordingly, the Parliament Q.9) Ans: c
enacted the CAG’s (Duties, Powers and Conditions Exp:
of Service) act, 1971. This Act was amended in • Option C is correct: The role of CAG is to uphold
1976 to separate accounts from audit in the the Constitution of India and the laws of
Central government. Parliament in the field of financial administration.

DPP 2023_DAY 34 16
Download More Test Series & eBooks in Hindi & English from our telegram channel- https://t.me/upsc_success_time1
Contact us :info@onlyias.com

OnlyIAS Nothing Else Visit :dpp.onlyias.in


Contact : +91-7007 931 912

• The accountability of the executive (i.e., council of


Example- Damodar Valley Corporation, Oil and
ministers) to the Parliament in the sphere of
Natural Gas Commission, Air India, Indian
financial administration is secured through audit
Airlines Corporation, and others.
reports of the CAG.
2. Some other corporations are audited by private
• The CAG is an agent of the Parliament and
professional auditors who are appointed by the
conducts audits of expenditure on behalf of the
Central Government in consultation with the
Parliament. Therefore, he is responsible only to
CAG. If necessary, the CAG can conduct a
the Parliament.
supplementary audit.
Example- Central Warehousing Corporation,
Q. 10) Ans: b
Industrial Finance Corporation, and others.
Exp:
3. Some other corporations are totally subjected
• Statement 1 is correct: The Constitution of India
to private audits.
visualizes the CAG to be Comptroller as well as
Example- RBI, SBI, FCI etc.
Auditor General. However, in practice, the CAG is
fulfilling the role of an Auditor-General only and
not that of a Comptroller. In other words, the CAG Q.12) Ans: c
has no control over the issue of money from the Exp:
consolidated fund and many departments are Model Code of Conduct
authorized to draw money by issuing cheques • The Model Code of Conduct (MCC) is a set of
without specific authority from the CAG, who is guidelines issued by the Election Commission of
concerned only at the audit stage when the India for conduct of political parties & candidates
expenditure has already taken place. during elections.
• Statement 2 is incorrect: In this respect, the CAG • The norms have been evolved with the consensus
of India differs totally from the CAG of Britain who of political parties who have consented to abide by
has powers of both Comptroller as well as Auditor the principles embodied in the said code and also
General. In other words, in Britain, the executive binds them to respect and observe it in it’s letter
can draw money from the public exchequer only and spirit.
with the approval of the CAG. • The Model Code also prescribes guidelines for the
ruling party either at the Center or in the State to
Q.11) Ans: a ensure that a level field is maintained and that no
Exp: cause is given for any complaint that the ruling
• Option A is correct: Some corporations are totally party has used its official position for the purposes
subjected to private audit. In other words, their of its election campaign.
audit is done exclusively by private professional • Statement 1 is incorrect: The model code of
auditors and the CAG does not come into the conduct cannot be enforceable by law because it
picture at all. is not statutory.
• They submit their annual reports and accounts • Statement 2 is incorrect: Model code of conduct is
directly to the Parliament. not mentioned anywhere in the constitution.
• Examples of such corporations are Life Insurance Therefore, it does not have constitutional
Corporation of India, Reserve Bank of India, State authenticity and nor is a source of constitutional
Bank of India, Food Corporation of India, and power of the Election Commission.
others. • Statement 3 is incorrect: No provision of model
code of conduct has been made in the RPAct,
1951.Thus, it is not statutorily binding.
Extra Edge by only IAS
The role of CAG in the auditing of public corporations
Q.13) Ans: d
is limited. Broadly speaking, his relationship with the
Exp:
public corporations falls into the following three
Electronic Voting Machine
categories:
• Electronic voting machine is a wireless, stand-
1. Some corporations are audited totally and
alone unit which works on a 'one -time
directly by the CAG.
Programmable chip'.
• Advantage of EVM:

DPP 2023_DAY 34 17
Download More Test Series & eBooks in Hindi & English from our telegram channel- https://t.me/upsc_success_time1
Contact us :info@onlyias.com

OnlyIAS Nothing Else Visit :dpp.onlyias.in


Contact : +91-7007 931 912

o Electronic Voting Machines have been developed Thus, they are not subject to the vote of
to facilitate easy polling and counting. Parliament.
o The use of the machine is to save the cost of paper • Further, no minister can represent the CAG in
and printing etc. Parliament (both Houses) and no minister can be
o To get the result within three to four hours, thus called upon to take any responsibility for any
saving a lot of manual exercise involved in actions done by him. Hence Option (b) is correct.
conventional counting.
• Statement 1 is incorrect: The machines have been Q.15) Ans: c
developed by Electronics Corporation of India Exp:
Limited (ECIL) and Bharat Electronics Corporation Comptroller and Auditor General of India
of India Limited (BEL). The Constitution (Article 149) authorizes the
• These machines provide full safeguard for ensuring Parliament to prescribe the duties and powers of the
secrecy of ballots and against tampering of CAG in relation to the accounts of the Union and of the
machines apart from ensuring rapidity of poll and states and of any other authority or body. The duties
instantaneous results. and functions of the CAG as laid down by the
• Statement 2 is incorrect: The Parliament has Parliament and the Constitution are:
amended the Representation of People Act, 1951, • He audits the accounts related to all expenditure
in March, 1989, introducing Section 61 (A) in the from the Consolidated Fund of India, consolidated
said Act, which provided for the recording of votes fund of each state and consolidated fund of each
of voting machines in such manner as may be union territory having a Legislative Assembly.
prescribed, may be adopted in such constituencies • He audits all expenditure from the Contingency
or constituencies as the Election Commission may Fund of India and the Public Account of India as
specify. well as the contingency fund of each state and the
• In pursuance of the above provisions, the Central public account of each state.
Government amended the Conduct of Election • He audits the receipts and expenditure of the
Rules, 1961, by inserting a new Chapter II [Rules Centre and each state to satisfy himself that the
49(a) to 49(x)] for facilitating the use of Electronic rules and procedures in that behalf are designed
Voting Machines. to secure an effective check on the assessment,
collection and proper allocation of revenue.
Q.14) Ans: b He audits the receipts and expenditure of the
Exp: following:
Comptroller and Auditor General of India o All bodies and authorities substantially financed
Article 148 of the Constitution of India provides for an from the Central or state revenues;
independent office of the Comptroller and Auditor o Government companies; and
General of India (CAG). The Constitution has made the o Other corporations and bodies, when so
following provisions to safeguard and ensure the required by related laws.
independence of CAG - • He audits all transactions of the Central and state
• He is provided with the security of tenure. He can governments related to debt, sinking funds,
be removed by the president only in accordance deposits, advances, suspense accounts and
with the procedure mentioned in the Constitution. remittance business. He also audits receipts, stock
Thus, he does not hold his office till the pleasure accounts and others, with approval of the
of the president, though he is appointed by him. President, or when required by the President.
• His salary and other service conditions are
determined by the Parliament. His salary is equal Q.16) Ans: a
to that of a judge of the Supreme Court. Exp:
• Neither his salary nor his rights in respect of leave • Statement 1 and 2 are correct: The Constitution
of absence, pension or age of retirement can be (Article 165) has provided for the office of the
altered to his disadvantage after his appointment. advocate general for the states. He is the highest
• The administrative expenses of the office of the law officer in the state. The advocate general is
CAG, including all salaries, allowances and appointed by the governor. The conditions for
pensions of persons serving in that office are appointing a Advocate General provided by the
charged upon the Consolidated Fund of India. Constitution are:

DPP 2023_DAY 34 18
Download More Test Series & eBooks in Hindi & English from our telegram channel- https://t.me/upsc_success_time1
Contact us :info@onlyias.com

OnlyIAS Nothing Else Visit :dpp.onlyias.in


Contact : +91-7007 931 912

o He must be a person who is qualified to be


Budget (including Railway Budget) to the
appointed a judge of a high court. In other words:
parliament and budget for the state
▪ he must be a citizen of India and must have held
Governments under President's Rule and union
a judicial office for ten years or been an
territory administrations.
advocate of a high court for ten years.
• NITI Aayog: NITI AYOG is a premiere policy think
• Statement 3 is incorrect: There is no such
tank of the Government of India. It was
condition provided by the Constitution for the
established with the aim to achieve sustainable
appointment of Advocate General of State.
development goals by active involvement of
Although the President of India can appoint an
state government in the planning process
eminent jurist in his opinion as the Attorney
o This premium policy think tank was
General of India.
established in 2015 via an executive resolution
by replacing the Planning Commission of India
Q.17) Ans: a
o NITI Aayog’s entire gamut of activities can be
Exp:
divided into four main heads:
Comptroller and Auditor General(CAG)
▪ Policy and Programme Framework
• The Constitution of India under Article 148
▪ Cooperative Federalism
provides for an independent office of Comptroller
▪ Monitoring and Evaluation
and Auditor General. He is the head of the Indian
▪ Think Tank, and Knowledge and Innovation
Audit and Accounts Department.
Hub.
• He is the guardian of the public purse and controls
the entire financial system of the country at both
the levels- the Centre and the State. Q.18) Ans: a
o CAG submits 3 reports to the President, who lays Exp:
these reports before both the Houses of • Statement a is correct: CAG by aiding legislature
Parliament. After this, the public accounts ensures the financial accountability of the
committee examines these reports. CAG acts as executive by scrutinising the financial transactions
a friend, guide and philosopher to the public of the Government and brings the results of such
accounts committee to examine these reports. scrutiny before the legislature.
o CAG ascertains and certifies the net proceeds of o There was an Auditor-General of India even under
any tax or duty(Article 279). His certificate is the Government of India Act, 1935 and that act
final. The 'net proceeds ' means the proceeds of secured the independence of the Auditor-General
a tax or duty minus the cost of collection. by making him irremovable except "in like manner
and of grounds as a Judge of Federal Court". So
Extra Edge by Only IAS
the office of CAG, in the Constitution is
• Finance Commission: Under Article 280 of the
substantially modelled upon that of Auditor-
Constitution, the President of India is required to
General under the Government of India Act, 1935.
constitute a Finance Commission at an interval
of five years or earlier.
Q.19) Ans: c
o The Finance Commission (FC) is a
Exp:
constitutional body, that determines the
• Statement 1 is incorrect: CAG duty is to uphold the
method and formula for distributing the tax
Constitution of India and laws of Parliament in the
proceeds between the Centre and states, and
field of financial administration.
among the states as per the constitutional
o He is provided with security of tenure. Although
arrangement and present requirements
he is appointed by the President by warrant
• Department of Economic Affairs: The
under his hand and seal but shall only be
Department of Economic Affairs is the nodal
removed from office in like manner and on the
agency of the Union Government to formulate
like grounds as a Judge of the Supreme Court.
and monitor a country's economic policies and
Thus, he does not hold his office till the pleasure
programmes having a bearing on domestic and
of the President, though he is appointed by him.
international aspects of economic management.
• Statement 2 is correct: The Constitution of India
A principal responsibility of this Department is
under Article 76 has provided for the office of the
the preparation and presentation of the Union

DPP 2023_DAY 34 19
Download More Test Series & eBooks in Hindi & English from our telegram channel- https://t.me/upsc_success_time1
Contact us :info@onlyias.com

OnlyIAS Nothing Else Visit :dpp.onlyias.in


Contact : +91-7007 931 912

Attorney General for India. He is the highest law can be altered to his disadvantage after his
officer in the country. appointment.
o The Attorney General(AG) is appointed by the o The administrative expenses of the office of the
President. The term of office of AG is not fixed by CAG, including all salaries, allowances and
the Constitution. Further, the Constitution does pensions of persons serving in that office are
not contain the procedure and grounds for his charged upon the Consolidated Fund of India.
removal. He holds office during the pleasure of Thus, they are not subject to the vote of
the President. Parliament.
o This means that he may be removed by the o The conditions of service of persons serving in
President at any time. He may also quit his office the Indian Audit and Accounts Department and
by submitting his resignation to the President. the administrative powers of the CAG are
• Statement 3 is incorrect: The Constitution under prescribed by the president after consultation
Article 165 has provided for the office of Advocate with the CAG.
General for the States. He is the highest law officer
in the States. Q.21) Ans: a
o The Advocate General is appointed by the Exp:
Governor. The term of office of Advocate • Statement 1 and 2 is correct: The CAG, before
General is not fixed by the Constitution. Further, taking over his office, makes and subscribes before
the Constitution does not contain the procedure the President or some person appointed in that
and grounds for his removal. He holds office behalf by him, an oath or affirmation according to
during the pleasure of the Governor. the form set out for the purpose in the 3rd
o This means that he may be removed by the Schedule of the Constitution:
Governor at any time. He may also quit his office o to bear true faith and allegiance to the
by submitting his resignation to the Governor. Constitution of India;
o to uphold the sovereignty and integrity of India;
Q.20) Ans: d o to duly and faithfully and to the best of his ability,
Exp: knowledge and judgement perform the duties of
• Statement 1 is incorrect: Comptroller and Auditor his office without fear or favour, affection or ill-
General of India is the apex authority responsible will;
for external and internal audits of the expenses of o to uphold the Constitution and the laws.
the National and state governments. The CAG • Statement 3 is incorrect: Schedule 3 of the
submits three audits reports to the President: Constitution enshrines the oath and affirmation
o Audit report on appropriation for:
o Audit report on finance accounts o Union Ministers of India and State Ministers
o ‘Audit report on public undertakings o Parliament Election Candidates and State
• The President lays these reports before both the legislature Election Candidates
Houses of the Parliament. After this, the Public o Members of Parliament and State legislature
Accounts Committee examines them and reports Members
its findings to the Parliament. o Supreme Court Judges and High Court Judges
o The appropriation accounts compare the actual o Comptroller and Auditor General
expenditure with the expenditure sanctioned by • To devote himself to the service and well-being of
the Parliament through the Appropriation Act. people is part of President and Governor's oath.
o The finance accounts show the annual receipts
and disbursements of the Union government. Q.22) Ans: d
• Statement 2 and 3 is incorrect: The Constitution Exp:
has made the various provisions to safeguard and • Statement 1 is correct: The Contempt of Courts
ensure the independence of CAG. Act, 1971, lays down the law on contempt of court.
o His salary and other service conditions are Section 15 of the legislation describes the
determined by the Parliament. His salary is equal procedure on how a case for contempt of court can
to that of a judge of the Supreme Court. be initiated:
o Neither his salary nor his rights in respect of o In the case of the Supreme Court, the Attorney
leave of absence, pension or age of retirement General or the Solicitor General, and in the case

DPP 2023_DAY 34 20
Download More Test Series & eBooks in Hindi & English from our telegram channel- https://t.me/upsc_success_time1
Contact us :info@onlyias.com

OnlyIAS Nothing Else Visit :dpp.onlyias.in


Contact : +91-7007 931 912

of High Courts, the Advocate General, may bring responsible only to the Parliament. But does not
in a motion before the court for initiating a case enjoy the privileges and immunities that are
of criminal contempt. available to the member of Parliament.
o However, if the motion is brought by any other • Statement 2 is correct: The Attorney General is the
person, the consent in writing of the Attorney highest law officer in the country. The Attorney
General or the Advocate General is required. General has the right of audience in all courts in
o The motion or reference made for initiating the the territory of India. Further, he has the right to
case will have to specify the contempt of which speak and to take part in the proceedings of both
the person charged is alleged to be guilty. the Houses of Parliament or their joint sitting and
o The objective behind requiring the consent of the any committee of the Parliament of which he may
Attorney General before taking cognizance of a be named a member, but without a right to vote.
complaint is to save the time of the court. Judicial He enjoys all the privileges and immunities that are
time is squandered if frivolous petitions are made available to a member of Parliament.
and the court is the first forum for bringing them • Statement 3 is incorrect: In addition to the Auditor
in. The AG’s consent is meant to be a safeguard General, there are other law officers of the
against frivolous petitions, as it is deemed that the Government of India. They are the solicitor general
AG, as an officer of the court, will independently of India and additional solicitor general of India.
ascertain whether the complaint is indeed valid. They assist the AG in the fulfilment of his official
The law has a limitation period of one year for responsibilities. It should be noted here that only
bringing in action against an individual. the office of the AG is created by the Constitution.
• Statement 2 is correct: Contempt of court is the In other words, Article 76 does not mention the
power of the court to protect its own majesty and solicitor general and additional solicitor general.
respect. The expression ‘contempt of court’ has They do not enjoy the privileges and immunities
not been defined by the Constitution. The that are available to a member of Parliament.
Contempt of Courts Act, 1971 defines both civil
Extra Edge By Only IAS
and criminal contempt.
Parliamentary Privileges
o Civil contempt refers to wilful disobedience to
• Parliamentary privileges are special rights,
any judgement of the court.
immunities and exemptions enjoyed by the two
o Criminal contempt can be invoked if an act:
Houses of Parliament, their committees and
o Tends to scandalise or lower the authority of the
their members. They are necessary in order to
court.
secure the independence and effectiveness of
o Tends to interfere with the due course of any
their actions.
judicial proceeding.
• The Constitution has also extended the
o Obstruct the administration of justice.
parliamentary privileges to those persons who
• Statement 3 is correct: Article 129 of the
are entitled to speak and take part in the
Constitution gives the Supreme Court the power
proceedings of a House of Parliament or any of
to initiate contempt cases on its own,
its committees. These include the attorney
independent of the motion brought before it by
general of India and Union ministers.
the AG or with the consent of the AG.
• “The Supreme Court shall be a court of record and
shall have all the powers of such a court including Q.24) Ans: d
the power to punish for contempt of itself. Exp:
• In 1991, the Supreme Court ruled that it has power • Statement 1 is incorrect: The Attorney General of
to punish for contempt not only of itself but also of India is not a member of cabinet as in case of the
high courts, subordinate courts and tribunals U.K government. He does enjoy parliamentary
functioning in the country. privileges. He has the right to speak and to take
part in the proceedings of both the Houses of
Q.23) Ans: b Parliament or their joint sitting and any committee
Exp: of the Parliament of which he may be named a
• Statement 1 is incorrect: The CAG is an agent of member, but without a right to vote.
Parliament and conducts audit of expenditure on • Statement 2 is incorrect: He should not advise any
behalf of the Parliament. Therefore, he is ministry or department of Government of India or

DPP 2023_DAY 34 21
Download More Test Series & eBooks in Hindi & English from our telegram channel- https://t.me/upsc_success_time1
Contact us :info@onlyias.com

OnlyIAS Nothing Else Visit :dpp.onlyias.in


Contact : +91-7007 931 912

any statutory organisation or any public sector • Statement 2 is correct: This is the reason why Dr B
undertaking unless the proposal or a reference in R Ambedkar said that the CAG shall be the most
this regard is received through the Ministry of Law important Officer under the Constitution of India.
and Justice, Department of Legal Affairs. He is one of the bulwarks of the democratic
system of government in India; the others being
Q.25) Ans: c the Supreme Court, the Election Commission and
Exp: the Union Public Service Commission.
• Statement 1 is incorrect: CAG is one of the
bulwarks of the democratic system of government Q.27) Ans: a
in India. Various provisions are provided in the Exp:
Constitution to ensure its independence. • Statement 1 is incorrect: The Constitution (Article
o He is provided with security of tenure. Although 165) has provided for the office of the advocate
he is appointed by the President by warrant under general for the states. He is the highest law officer
his hand and seal but shall only be removed from in the state. Thus, he corresponds to the Attorney
office in like manner and on the like grounds as a General of India. The advocate general is
Judge of Supreme Court. Thus, he does not hold appointed by the governor. He must be a person
his office till the pleasure of the President, though who is qualified to be appointed a judge of a high
he is appointed by him. court. In other words, he must be a citizen of India
• Statement 2 is correct: To ensure the and must have held a judicial office for ten years or
independence of office of CAG, the Constitution been an advocate of a high court for ten years
provides that he shall not be eligible for further • Statement 2 and 3 are correct: The term of office
office either under the Government of India or of the advocate general is not fixed by the
under the Government of any State after he has Constitution. Further, the Constitution does not
ceased to hold his office, so that he shall have no contain the procedure and grounds for his
inducement to please the Executive of the Union removal. He holds office during the pleasure of the
or of any State. governor. This means that he may be removed by
• Statement 3 is incorrect: His salary and other the governor at any time. He may also quit his
service conditions are determined by the office by submitting his resignation to the
Parliament. Under this power, Parliament has governor. Conventionally, he resigns when the
enacted the Comptroller and Auditor-General's government (council of ministers) resigns or is
(Conditions of Service) Act, 1971. According to replaced, as he is appointed on its advice.
which:
o His salary shall be equal to that of a Judge of Q.28) Ans: d
Supreme Court. Exp:
o In matter of conditions of service, shall be • Statement 1 is correct: The Advocate General is
determined by the Rules application to the appointed by the governor. He must be a person
member of I.A.S., holding the rank of Secretary to who is qualified to be appointed a judge of a high
the Government of India. court, It means, he must be a citizen of India and
must have held a judicial office for ten years or
Q.26) Ans: b been an advocate of a high court for ten years
Exp: • Statement 2 and 3 are correct: The term of office
• Statement 1 is incorrect: Article 148 of the Indian of the AG is not fixed by the Constitution. The
constitution provides for an independent office of Constitution also does not contain the procedure
the Comptroller and Auditor General of India and grounds for his removal. He holds office during
(CAG). He is the head of the Indian Audit and the pleasure of the governor. It means He may be
Accounts. He is the guardian of the public purse removed by the governor at any time. He may also
and controls the entire financial system of the quit his office by submitting his resignation to the
country at both the levels—the Center and the governor. Conventionally, he resigns when the
state. His duty is to uphold the Constitution of government (council of ministers) resigns or is
India and laws of Parliament in the field of financial replaced, as he is appointed on its advice. The
administration. remuneration of the advocate general is not fixed

DPP 2023_DAY 34 22
Download More Test Series & eBooks in Hindi & English from our telegram channel- https://t.me/upsc_success_time1
Contact us :info@onlyias.com

OnlyIAS Nothing Else Visit :dpp.onlyias.in


Contact : +91-7007 931 912

by the Constitution. He receives such Exp:


remuneration as the governor may determine. The Finance Commission consists of a chairman and
four other members to be appointed by the president.
Q.29) Ans: c The qualification given in the question is about the
Exp: members of the Commission and not the Chairman.
• Statement 1 is correct: The Constitution (Article Qualification is different for members and the
165) has provided for the office of the Advocate Chairman.
General for the states. He is the highest law officer • The Qualification for Chairman: The chairman
in the state. As the chief law officer of the should be a person having experience in public
government in the state, the duties of the advocate affairs. (Nothing Else)
general include the following: • The qualification for members of the
o To give advice to the government of the state commission:
upon such legal matters which are referred to 1. A judge of the high court or one qualified to be
him by the governor. appointed as one.
o To perform such other duties of a legal character 2. A person who has specialized knowledge of finance
that are assigned to him by the governor. and accounts of the government.
o To discharge the functions conferred on him by 3. A person who has wide experience in financial
the Constitution or any other laws. matters and in Administration.
• Statement 2 is incorrect: In the performance of his 4. A person who has special knowledge of economics.
official duties, the advocate general is entitled to
appear before any court of law within the state. Q.32) Ans: a
Further, he has the right to speak and to take part Exp:
in the proceedings of both the Houses of the state • Statement 1 is correct: The Finance Commission is
legislature or any committee of the state required to make recommendations to the
legislature of which he may be named a member, President of India on the following matters:
but without a right to vote. 1. The distribution of the net proceeds of taxes to
• Statement 3 is correct: He enjoys all the privileges be shared between the Centre and the states,
and immunities that are available to a member of and the allocation between the states of the
the state legislature. respective shares of such proceeds.
2. The principles that should govern the grants-
Q.30) Ans: c in-aid to the states by the Centre (i.e., out of
Exp: the consolidated fund of India).
• Statement 1 is incorrect: Article 280 of the 3. The measures needed to augment the
Constitution of India provides for a Finance consolidated fund of a state to supplement the
Commission as a quasi-judicial body. It is resources of the panchayats and the
constituted by the president of India every fifth municipalities in the state on the basis of the
year or at such an earlier time as he considers recommendations made by the state finance
necessary. commission.
• Statement 2 is correct: The Finance Commission 4. Any other matter referred to it by the
consists of a chairman and four other members to president in the interests of sound finance.
be appointed by the president. They hold office for • Statement 2 is incorrect: The finance commission
such a period as specified by the president in his is also vested with the power to determine the
order. They are eligible for reappointment. principles that should govern the grants-in-aid to
• Statement 3 is incorrect: The Constitution the states by the Centre (i.e., out of the
authorizes the Parliament to determine the consolidated fund of India).
qualifications of members of the commission and
the manner in which they should be selected. SO Q.33) Ans: d
It’s the PARLIAMENT and not the president who Exp:
determines the qualification of the Chairman and • Statement 1 is correct: The Finance Commission is
Members of the Finance Commission. required to make recommendations to the
president of India on the following matters:
Q.31) Ans: d

DPP 2023_DAY 34 23
Download More Test Series & eBooks in Hindi & English from our telegram channel- https://t.me/upsc_success_time1
Contact us :info@onlyias.com

OnlyIAS Nothing Else Visit :dpp.onlyias.in


Contact : +91-7007 931 912

o The distribution of the net proceeds of taxes to be Exp:


shared between the Centre and the states, and the All statements are correct:
allocation between the states of the respective • Article 280 of the Constitution of India provides for
shares of such proceeds. a Finance Commission as a quasi-judicial body.
o The principles that should govern the grants-in-aid • It is constituted by the president of India every
to the states by the Centre (i.e., out of the fifth year or at such an earlier time as he considers
consolidated fund of India). necessary.
o The measures needed to augment the • The Finance Commission consists of a chairman
consolidated fund of a state to supplement the and four other members.
resources of the panchayats and the municipalities • Chairman and all other members of commission to
in the state on the basis of the recommendations be appointed by the president.
made by the state finance commission • They hold office for such a period as specified by
o Any other matter referred to it by the president in the president in his order.
the interests of sound finance. • They are eligible for reappointment.
• Statement 2nd is incorrect: The
recommendations made by the Finance Q.36) Ans: d
Commission are only of advisory nature and Exp:
hence, not binding on the government. It is up to All statements are correct:
the Union government to implement its • The Constitution authorizes the Parliament to
recommendations on granting money to the determine the qualifications of members of the
states. To put it in other words, ‘It is nowhere laid commission and the manner in which they should
down in the Constitution that the be selected.
recommendations of the commission shall be • Accordingly, the Parliament has enacted the
binding upon the Government of India or that it Finance Commission Act, 1951 to specify the
would give rise to a legal right in favour of the qualifications of the chairman and members of the
beneficiary states to receive the money commission.
recommended to be offered to them by the • The Finance Commission consists of a chairman
Commission’. and four other members to be appointed by the
• Statement 3rd is incorrect: Recommendations of president. They hold office for such a period as
the finance commission are applicable to both specified by the president in his order. They are
Union Government and State Governments. The eligible for reappointment.
state finance commission recommends devolution
of the net proceeds of taxes to the Panchayats. Q.37) Ans: a
Exp:
Q.34) Ans: a • Statement 1 is correct: The Parliament has
Exp: specified the qualifications of the chairman and
The finance commission of India members of the commission. The chairman should
• The Finance commission of India is a Quasi-judicial be a person having experience in public affairs.
and Constitutional body. Article 280 of the • Statement 2 is incorrect: The four other members
Constitution of India provides for a Finance should be selected from amongst the following:
Commission as a quasi-judicial body. o A judge of the high court or one qualified to be
• It is constituted by the president of India every fifth appointed as one.
year or at such an earlier time as he considers o A person who has specialized knowledge of
necessary. finance and accounts of the government.
• The Constitution authorizes the Parliament to o A person who has wide experience in financial
determine the qualifications of members of the matters and in administration.
commission and the manner in which they should o A person who has special knowledge of
be selected. Accordingly, the Parliament has economics.
specified the qualifications of the chairman and
members of the commission. Q.38) Ans: a
Exp:
Q.35) Ans: d

DPP 2023_DAY 34 24
Download More Test Series & eBooks in Hindi & English from our telegram channel- https://t.me/upsc_success_time1
Contact us :info@onlyias.com

OnlyIAS Nothing Else Visit :dpp.onlyias.in


Contact : +91-7007 931 912

• The Finance Commission is a Constitutionally qualifications of members of the commission and


mandated body that is at the centre of fiscal the manner in which they should be selected.
federalism. Accordingly, the Parliament has enacted the
• Set up under Article 280 of the Constitution, its Finance Commission Act, 1951 to specify the
core responsibility is to evaluate the state of qualifications of the chairman and members of the
finances of the Union and State Governments, commission.
recommend the sharing of taxes between them, • Statement 4 is NOT correct: The Finance
and lay down the principles determining the Commission consists of a chairman and four other
distribution of these taxes among States. members. Chairman and all other members of
• The Finance commission submits its report to the commission to be appointed by the president. They
president. He lays it before both the Houses of hold office for such a period as specified by the
Parliament along with an explanatory president in his order. They are eligible for
memorandum as to the action taken on its reappointment.
recommendations.
Q.40) Ans: b
Exp:
Extra edge by only IAS
• Statement 1 is Not correct: In present time finance
S. No. Commission —------------------------------------
commission not giving any suggestion on grants
Report
given to the States of Assam, Bihar, Odisha and
1. Union Public Service
West Bengal in lieu of assignment of any share of
Commission President
the net proceeds in each year of export duty on
2. Finance commission President
jute and jute products. The Finance Commission
3. Comptroller and Auditor General of
used to give suggestions on this issue till 1960.
India President (Article-151)
These grants were to be given for a temporary
4. National Human Right
period of ten years from the commencement of
Commission Central Govt. and
the Constitution.
concerned state govt.
• Statement 2 and 3 is correct: The Finance
Commission is required to make recommendations
Q.39) Ans: d to the president of India on the following matters:
Exp: o The distribution of the net proceeds of taxes to be
• Statement 1 is correct: The recommendations shared between the Centre and the states, and
made by the Finance Commission are only of the allocation between the states of the
advisory nature and hence, not binding on the respective shares of such proceeds.
government. It is up to the Union government to o The principles that should govern the grants-in-
implement its recommendations on granting aid to the states by the Centre (i.e., out of the
money to the states. To put it in other words,‘It is consolidated fund of India).
nowhere laid down in the Constitution that the o The measures needed to augment the
recommendations of the commission shall be consolidated fund of a state to supplement the
binding upon the Government of India or that it resources of the panchayats and the
would give rise to a legal right in favour of the municipalities in the state on the basis of the
beneficiary states to receive the money recommendations made by the state finance
recommended to be offered to them by the commission.
Commission. o Any other matter referred to it by the president in
• Statement 2 is correct: The constitution of India the interests of sound finance.
envisages the Finance commission as the balancing
wheel of fiscal federalism in India. However, its Q.41) Ans: d
role in the Centre–state fiscal relations was Exp:
undermined by the emergence of the erstwhile • Statement 1 is correct: Composition of 'National
Planning Commission, a non-constitutional and a Commission For Backward Class':
non-statutory body. a) Chairperson
• Statement 3 is correct: The Constitution b) Vice-Chairperson
authorises the Parliament to determine the c) Three other Members

DPP 2023_DAY 34 25
Download More Test Series & eBooks in Hindi & English from our telegram channel- https://t.me/upsc_success_time1
Contact us :info@onlyias.com

OnlyIAS Nothing Else Visit :dpp.onlyias.in


Contact : +91-7007 931 912

Representation of at least two persons having and other laws are being implemented and also
special knowledge related to backward classes evaluate how effective these safeguards are.
and one woman in NCBC is a welcome move which • Transparency: Article 342(A) introduces greater
makes the commission more democratic and transparency since it is mandatory to take
effective in promoting the interests of SEBCs. concurrence with Parliament for adding or
• Statement 2 is correct: Members are appointed by deleting any community in the backward classes
the President by warrant under his hand and seal. list.
The conditions of service and tenure of office of • Statement 2 is correct: The New NCBC can
the Chairperson, Vice-Chairperson and other effectively redress the grievances of backward
Members is determined by the President. classes with the power of the civil court. The Act
• Statement 3 is correct: The commission presents brings NCBC on par with the National Commission
an annual report to the President. The President for SCs and National Commission for STs.
places all such reports before the Parliament, along
with a memorandum explaining the action taken Q.44) Ans: d
on the recommendations made by the Exp:
Commission. The President also forwards any • Statement 1 is correct: Constitutional
report of the Commission pertaining to a State Amendment Act, 100th: The President of India
Govt. to the state Governor. The Governor places Pranab Mukherjee gave his assent to the
it before the state legislature. Constitution (119th Amendment) Bill, 2013 that
related to the Land Boundary Agreement (LBA)
Q.42) Ans: d between India and Bangladesh.
Exp: • Statement 2 is correct: Constitutional
• In 2018, 102nd Constitution Amendment Act Amendment Act, 101st: Introduced the Goods and
passed by parliament to provide constitutional Services Tax in the country from 1 July 2017. The
status to the National Commission for Backward Goods and Services Tax (GST) is a value-added tax.
Classes (NCBC). GST is levied on most goods and services sold for
• Objective: To oversee the implementation of domestic consumption. The GST is paid by
various safeguards provided to Backward Class consumers, but it is remitted to the government.
under the Constitution or under any other law for • Statement 3 is correct: Constitutional
time being in force or under any other order to the Amendment Act, 102nd: In 2018, 102nd
Govt. and to evaluate the working of such Constitution Amendment Act passed by
safeguards. parliament to provide constitutional status to the
• The 102nd amendment act has recognized that National Commission for Backward Classes
backward classes need holistic development and (NCBC). This Commission has the power to
equality in all parameters in addition to examine complaints and welfare measures
reservations. regarding socially and educationally backward
• Therefore, it has provisions for the development of classes.
Socially and Educationally Backward Classes • Statement 4 is correct: Constitutional
(SEBC) and the new NCBC’s role in their Amendment Act, 103rd: The Act amends Articles
development process. 15 and 16 of the Constitution — ensuring
Fundamental Rights to a citizen — by adding a
Q.43) Ans: c clause that allows the State to make “special
Exp: provision for the advancement of any
• Statement 1 is correct: NCBC would not encroach economically weaker sections of citizens”. These
upon the rights of the State govt. as they would provisions would relate to “their admission to
have their own backward class commissions and educational institutions, including private
their own lists of castes who belong to OBC. The educational institutions, whether aided or unaided
NCBC would recommend only to the central govt. by the State, other than the minority educational
regarding inclusion or deletion of a particular caste institutions”.
in the list. It will ensure the commission will
investigate and monitor how safeguards provided Q.45) Ans: c
to the Backward Classes under the Constitution Exp:

DPP 2023_DAY 34 26
Download More Test Series & eBooks in Hindi & English from our telegram channel- https://t.me/upsc_success_time1
Contact us :info@onlyias.com

OnlyIAS Nothing Else Visit :dpp.onlyias.in


Contact : +91-7007 931 912

About NCSC
• Regular reporting to the President of the country
• NCSC is a constitutional body that works to
on the implementation of these safeguards.
safeguard the interests of the scheduled castes
• Recommending steps to be taken to further the
(SC) in India. Article 338 of the constitution of India
socio-economic development and other welfare
deals with this commission. It provides for a
activities of the SCs.
National Commission for the Scheduled Castes and
• Any other function with respect to the welfare,
Scheduled Tribes .
protection, development and advancement of
Duties and Functions of the Commission:
the SC community.
• Statement 1 is correct: To investigate and monitor
• The Commission is also required to discharge
all matters relating to the safeguards provided for
similar functions with regard to the Anglo-Indian
the Scheduled Castes under the Constitution or
Community as it does with respect to the SCs.
under any other law for the time being in force or
under any order of the Government and to
evaluate the working of such safeguards; Q.46) Ans: d
• To inquire into specific complaints with respect to Exp:
the deprivation of rights and safeguards of the Powers of the Commission:
Scheduled Castes; • Summoning and enforcing the attendance of any
• To participate and advise in the planning process of person from any part of India and examining him
socio-economic development of the Scheduled on oath.
Castes and to evaluate the progress of their • Requiring the discovery and production of any
development under the Union and any State; documents.
• Statement 2 is incorrect: To present to the • Receiving evidence on affidavits.
President, annually and at such other times as the • Requisitioning any public record or copy thereof
Commission may deem fit, reports upon the from any court or office.
working of those safeguards • Issuing summons/communications for the ex
• To make in such reports, recommendations as to amination of witnesses and documents.
the measures that should be taken by the Union or • Any
any State for effective implementation of those other matter which the President may by rule
safeguards and other measures for the protection, determine.
welfare and socio-economic development of the Enquiring into complaints relating to the
Scheduled Castes and deprivation of the rights and safeguards of the SCs.
• Statement 3 is correct: To discharge such other • Taking part in and advising the central or state
functions in relation to the protection, welfare and governments with respect to the planning of socio-
development and advancement of the Scheduled economic development of the SCs.
Castes as the President may, subject to the
provisions of any law made by Parliament, by rule Q.47) Ans: c
specify. Exp:
• Statement 1 is incorrect: The Constitution does
not define the criteria for recognition of Scheduled
Extra Edge by Only IAS
Tribes and hence the definition contained in 1931
Census was used in the initial years after
Functions:
independence. However, Article 366(25) of the
• Monitoring and investigating all issues
Constitution only provides process to define
concerning the safeguards provided for the SCs
Scheduled Tribes: “Scheduled Tribes means such
under the constitution.
tribes or tribal communities or parts of or groups
• Enquiring into complaints relating to the
within such tribes or tribal communities as are
deprivation of the rights and safeguards of the
deemed under Article 342 to be Scheduled Tribes
SCs.
for the purposes of this Constitution.”
• Taking part in and advising the central or state
• Statement 2 is incorrect: The largest number of
governments with respect to the planning of
tribal communities (62) are found in Odisha.
socio-economic development of the SCs.
• No Tribe was identified in Haryana, Punjab,
Chandigarh, Delhi, and Pondicherry

DPP 2023_DAY 34 27
Download More Test Series & eBooks in Hindi & English from our telegram channel- https://t.me/upsc_success_time1
Contact us :info@onlyias.com

OnlyIAS Nothing Else Visit :dpp.onlyias.in


Contact : +91-7007 931 912

Q.48) Ans: b directly established by Article 338 of the


Exp: Constitution.
• NCSC is a constitutional body which works to • The functions of the Commission are:
safeguard the interests of the scheduled castes o To investigate and monitor all matters relating to
(SC) in India. the constitutional and other legal safeguards for
• Article 338 of the constitution of India deals with the SCs and to evaluate their working; Hence,
this commission. It provides for a National Statement 1 is correct.
Commission for the Scheduled Castes and o To inquire into specific complaints with respect to
Scheduled Tribes with duties to investigate and the deprivation of rights and safeguards of the
monitor all matters relating to safeguards SCs;
provided for them, to inquire into specific o To participate and advise on the planning process
complaints and to participate and advise on the of socio-economic development of the SCs and to
planning process of their socio-economic evaluate the progress of their development under
development etc the Union or a state; Hence, Statement 2 is
• The NCBC is required to present annual reports to correct.
the President on working of the safeguards for o To present to the President, annually and at such
backward classes. These reports will be tabled in other times as it may deem fit, reports upon the
Parliament, and in the state legislative assemblies working of those safeguards;
of the concerned states. o To discharge such other functions in relation to
the protection, welfare and development and
Q.49) Ans: a advancement of the SCs as the president may
Exp: specify.
• The National Commission for Scheduled Castes
(SCs) is a constitutional body in the sense that it is Q.51) Ans: b
directly established by Article 338 of the Exp:
Constitution. On the other hand, the other • The separate National Commission for SCs came
national commissions like the National into existence in 2004. It consists of a chairperson,
Commission for Women (1992), the National a vice-chairperson and three other members. They
Commission for Minorities (1993), the National are appointed by the President by warrant under
Commission for Backward Classes (1993), the his hand and seal. Their conditions of service and
National Human Rights Commission (1993) and the tenure of office are also determined by the
National Commission for Protection of Child President.
Rights (2007) are statutory bodies in the sense that • Statement 1 is incorrect: The commission presents
they are established by acts of the Parliament. an annual report to the president. It can also
• The 89th Constitutional Amendment Act of 2003 submit a report as and when it thinks necessary.
bifurcated the combined National Commission for The President places all such reports before the
SCs and STs into two separate bodies, namely, Parliament, along with a memorandum explaining
National Commission for Scheduled Castes (under the action taken on the recommendations made
Article 338) and National Commission for by the Commission. The memorandum should also
Scheduled Tribes (under Article 338-A). contain the reasons for the non-acceptance of any
• The separate National Commission for SCs came of such recommendations.
into existence in 2004. It consists of a chairperson, • Statement 2 is correct: The President also
a vice-chairperson and three other members. They forwards any report of the Commission pertaining
are appointed by the President by warrant under to the state government to the state governor.
his hand and seal. Their conditions of service and The governor places it before the state legislature,
tenure of office are also determined by the along with a memorandum explaining the action
President. Hence option (A) is correct. taken on the recommendations of the
Commission. The memorandum should also
Q.50) Ans: c contain the reasons for the non-acceptance of any
Exp: of such recommendations.
• The National Commission for Scheduled Castes
(SCs) is a constitutional body in the sense that it is Q.52) Ans: c

DPP 2023_DAY 34 28
Download More Test Series & eBooks in Hindi & English from our telegram channel- https://t.me/upsc_success_time1
Contact us :info@onlyias.com

OnlyIAS Nothing Else Visit :dpp.onlyias.in


Contact : +91-7007 931 912

Exp: along with a memorandum explaining the action


• The National Commission for Scheduled Caste is taken on the recommendations of the
vested with the power to regulate its own Commission. The memorandum should also
procedure. The Commission, while investigating contain the reasons for the non-acceptance of any
any matter or inquiring into any complaint, has all of such recommendations.
the powers of a civil court trying a suit and in
particular in respect of the following matters: Q.54) Ans: b
a) summoning and enforcing the attendance of any Exp:
person from any part of India and examining him • Statement 1 is incorrect: The 89th Constitutional
on oath; Hence Statement 1 is correct. Amendment Act of 2003 bifurcated the combined
b) requiring the discovery and production of any National Commission for SCs and STs into two
document; separate bodies, namely, National Commission for
c) receiving evidence on affidavits; Scheduled Castes (under Article 338) and National
d) requisitioning any public record from any court or Commission for Scheduled Tribes (under Article
office; 338-A).
e) issuing summons for the examination of witnesses • The Commission is vested with the power to
and documents; and regulate its own procedure. The Commission,
f) any other matter which the President may while investigating any matter or inquiring into
determine. any complaint, has all the powers of a civil court
• The Central government and the state trying a suit and in particular in respect of the
governments are required to consult the following matters:
Commission on all major policy matters affecting i. summoning and enforcing the attendance of
the SCs. any person from any part of India and
• The Commission is also required to discharge examining him on oath;
similar functions with regard to the other ii. requiring the discovery and production of any
backward classes (OBCs) and the Anglo-Indian document;
Community as it does with respect to the SCs. In iii. receiving evidence on affidavits;
other words, the Commission has to investigate all iv. requisitioning any public record from any
matters relating to the constitutional and other court or office; Hence statement 3 is correct
legal safeguards for the OBCs and the Anglo-Indian v. issuing summons for the examination of
Community and report to the President upon their witnesses and documents; and Hence
working. statement 2 is correct.
vi. any other matter which the President may
Q.53) Ans: a determine.
Exp: The Central government and the state
• Statement 1 is correct: The National Commission governments are required to consult the
for Scheduled Castes (SCs) is a constitutional body Commission on all major policy matters
in the sense that it is directly established by Article affecting the SCs.
338 of the Constitution. One of the functions of • The Commission is also required to discharge
the Commission is to present to the President, similar functions with regard to the other
annually and at such other times as it may deem backward classes (OBCs) and the Anglo-Indian
fit, reports upon the working of those safeguards. Community as it does with respect to the SCs.
• The President places all such reports before the
Parliament, along with a memorandum explaining Q.55) Ans: b
the action taken on the recommendations made by Exp:
the Commission. The memorandum should also National Commission for Scheduled Castes
contain the reasons for the non-acceptance of any • National Commission for Scheduled castes (NCSC)
of such recommendations. is a Constitutional Body established by article 338
• Statement 2 is incorrect: The President also of the Constitution. The separate NCSC came into
forwards any report of the Commission pertaining existence in 2004. The 89th Constitutional
to a state government to the state governor. The amendment act of 2003 bifurcated it from the
governor places it before the state legislature, National Commission for Scheduled Tribes. It

DPP 2023_DAY 34 29
Download More Test Series & eBooks in Hindi & English from our telegram channel- https://t.me/upsc_success_time1
Contact us :info@onlyias.com

OnlyIAS Nothing Else Visit :dpp.onlyias.in


Contact : +91-7007 931 912

consists of Chairperson, vice-chairperson and three other members. They are appointed by the
three other members. They are appointed by the President by warrant under his hand and seal.
President by warrant under his hand. Their conditions of service and tenure of office are
• Statement 1 is correct: It provides safeguards also determined by the President.
against the exploitation of Scheduled Castes and
Anglo-Indian communities. It aims to promote and Q.57) Ans: a
protect their social, educational, economic and Exp:
cultural interests provided in the Constitution. It • Like the National Commission for Scheduled Castes
also participates in the planning process of the (SCs), the National Commission for Scheduled
socio-economic development of SCs and Anglo Tribes (STs) is also a constitutional body in the
Indians. sense that it is directly established by Article 338-A
• Statement 2 is not correct: The President specifies of the Constitution. In 2005, the President
what castes in each state and union territory are specified the following other functions of the
to be treated as the SCs. But any inclusion or Commission in relation to the protection, welfare
exclusion of any caste from presidential and development and advancement of the STs:
notification can be done by parliament only. NCSC o Measures to be taken over conferring ownership
has an advisory role only in this regard. rights in respect of minor forest produce to STs
• The National Commission for Scheduled Caste living in forest areas. Hence, statement 1 is
presents to the President, annually and at such correct.
other times as it may deem fit, reports upon o Measures to be taken to safeguard rights of the
safeguards of SCs and Anglo Indians. It also tribal communities over mineral resources, water
discharges such other functions in relation to the resources etc., as per law
protection, welfare and development of SCs as the o Measures to be taken for the development of
President may specify. tribals and to work for more viable livelihood
strategies
Q.56) Ans: c o Measures to be taken to improve the efficacy of
Exp: relief and rehabilitation measures for tribal
• Statement 1 is correct: Like the National groups displaced by development projects
Commission for Scheduled Castes (SCs), the o Measures to be taken to prevent alienation of
National Commission for Scheduled Tribes (STs) is tribal people from land and to effectively
also a constitutional body in the sense that it is rehabilitate such people in whose case alienation
directly established by Article 338-A of the has already taken place. Hence statement 3 is
Constitution. The National Commission for SCs and correct.
STs came into being consequent upon passing of o Measures to be taken to elicit maximum
the 65th Constitutional Amendment Act of 1990. cooperation and involvement of tribal
The Commission was established under Article 338 communities for protecting forests and
of the Constitution with the objective of undertaking social afforestation
monitoring all the safeguards provided for the SCs o Measures to be taken to ensure full
and STs under the Constitution or other laws. implementation of the Provisions of Panchayats
• Statement 2 is incorrect: Geographically and (Extension to the Scheduled Areas) Act, 1996.
culturally, the STs are different from the SCs and o Measures to be taken to reduce and ultimately
their problems are also different from those of eliminate the practice of shifting cultivation by
SCs. In 1999, a new Ministry of Tribal Affairs was tribals that lead to their continuous
created to provide a sharp focus to the welfare and disempowerment and degradation of land and
development of the STs. It was felt necessary that the environment.
the Ministry of Tribal Affairs should coordinate all
activities relating to the STs as it would not be Q.58) Ans: c
administratively feasible for the Ministry of Social Exp:
Justice and Empowerment to perform this role. • The Commission is vested with the power to
• Statement 3 is incorrect: The separate National regulate its own procedure. The Commission, while
Commission for STs came into existence in 2004. It investigating any matter or inquiring into any
consists of a chairperson, a vice-chairperson and complaint, has all the powers of a civil court trying

DPP 2023_DAY 34 30
Download More Test Series & eBooks in Hindi & English from our telegram channel- https://t.me/upsc_success_time1
Contact us :info@onlyias.com

OnlyIAS Nothing Else Visit :dpp.onlyias.in


Contact : +91-7007 931 912

a suit and in particular in respect of the following functions of SPSC are provided in Part XIV of the
matters: Indian Constitution under Article 315 to Article
o summoning and enforcing the attendance of any 323. The Chairman and other members of the
person from any part of India and examining him SPSC are appointed by the Governor of the State.
on oath; Hence statement 2 is correct. • Statement 2 is Incorrect: The Chairman or any
o requiring the discovery and production of any other member of SPSC shall only be removed from
document; his/her office by order of the President of India.
o receiving evidence on affidavits; The Governor of the state shall suspend the
o requisitioning any public record from any court Chairman or any other member from his/her office
or office; in respect of whom a reference has been made to
o issuing summons for the examination of the Supreme Court.The conditions for removal of
witnesses and documents, hence statement (1) members are the same as those of the UPSC.
is correct.
o any other matter which the President may Q.61) Ans: b
determine. Exp:
• The Central government and the state • Statement 1 is incorrect: Article 315 of the Indian
governments are required to consult the Constitution, two or more States may agree that
Commission on all major policy matters affecting there shall be one Public Service Commission for
the STs. that group of States. The resolution to such an
agreement shall be passed by each House of the
Q.59) Ans: a Legislature of each of the States. Only then, the
Exp: Parliament may by law provide for the
• The National Commission for Scheduled Castes appointment of a Joint State Public Service
(SCs), the National Commission for Scheduled Commission (JSPSC).
Tribes (STs) is also a constitutional body in the • Statement 2 is correct: Article 316 of the Indian
sense that it is directly established by Article 338- Constitution states that the Chairman and other
A of the Constitution. members of JSPSC shall be appointed by the
• Geographically and culturally, the STs are President of India. A member of Joint Commission
different from the SCs and their problems are also shall hold the office for a term of six years or till the
different from those of SCs. In 1999, a new Ministry age of 62 years, whichever is earlier
of Tribal Affairs was created to provide a sharp • Statement 3 is correct: Article 323 states that it
focus to the welfare and development of the STs. shall be the duty of the JSPSC to present the
• It was felt necessary that the Ministry of Tribal annual report to the Governor of each of the
Affairs should coordinate all activities relating to States which have together formed the
the STs as it would not be administratively feasible Commission. The Governor of each of the states is
for the Ministry of Social Justice and responsible for providing a memorandum
Empowerment to perform this role. explaining the cases the advice of the Commission
• The separate National Commission for STs came was not accepted. The reasons for non-acceptance
into existence in 2004. It consists of a chairperson, of advice of Commission are to be laid before the
a vice-chairperson and three other members. They Legislature of each of the States.
are appointed by the President by warrant under
his hand and seal. Their conditions of service and Q.62) Ans: a
tenure of office are also determined by the Exp:
President. Hence option (A) is correct. • Statement 1 is correct: The Union Public Service
Commission (UPSC) is the central recruiting agency
Q.60) Ans: a in India. It is an independent constitutional body.
Exp: The provisions regarding the composition of UPSC,
• Statement 1 is correct: Quite Similar to the UPSC appointment and removal of its members and the
at the Centre, there is a State Public Service powers and functions of UPSC are provided in Part
Commission (SPSC) in the state. The provisions XIV of the Indian Constitution under Article 315 to
regarding the composition of SPSC, appointment Article 323. The UPSC is consulted in making
and removal of its members and the powers and appointments to civil services and posts and in

DPP 2023_DAY 34 31
Download More Test Series & eBooks in Hindi & English from our telegram channel- https://t.me/upsc_success_time1
Contact us :info@onlyias.com

OnlyIAS Nothing Else Visit :dpp.onlyias.in


Contact : +91-7007 931 912

promotions and transfers from one service to • Statement 1 is correct: The provisions regarding
another depending upon the suitability of the composition of UPSC, appointment and
candidates. removal of its members and the powers and
• Statement 2 is correct: It shall be the duty of a functions of UPSC are provided in Part XIV of the
Public Service Commission to advise on any matter Indian Constitution under Article 315 to Article
referred to them by the President of India. Also 323. All the members of the commission are
UPSC is consulted while looking at all disciplinary appointed by the President of India with at least
matters affecting a person serving under the half of the members being the Civil Servants
Government of India or the Government of a (working or retired) with no less than ten years of
State in a civil capacity, including memorials or experience in Central or State service.
petitions relating to such matter. • Statement 2 is Incorrect: Article 318 of the
• Statement 3 is Incorrect: In order to exempt some constitution of India states that the Union Public
posts which for reasons of National Security or Service Commission will be constituted with a
some other reasons may not be required to be chairman and a fixed number of members; the
referred to the Commission for their advice, the number of such members and the terms and
Union Public Service Commission (Exemption from conditions of their service is to be determined by
Consultations) Regulations were issued on the President of India. The President, as such,
September 1, 1958, under Article 320(3)(a) and (b) appoints the Chairman and other members of the
of the Constitution. In line with this,UPSC is not commission for a period of six years. Hence there
consulted while making reservations of is no provision of Recommendation by two houses
appointments or posts in favour of any backward to the President. President of India is the final
class of citizens,as well as taking into authority in Appointment matters of Chairman and
consideration the claims of scheduled castes and members of the Commission.
scheduled tribes in making appointments to
services and posts. Q.65) Ans: d
Exp:
Q. 63) Ans: c The Union Public Service Commission
Exp: • UPSC is the central recruiting agency in India. It is an
• Statement 2 is correct: The Union Public Service independent constitutional body.The provisions
Commission of India was formed by the British regarding the composition of UPSC, appointment
Government during the British rule. In 1924, Lee and removal of its members and the powers and
Commission had suggested in its report for the functions of UPSC are provided in Part XIV of the
establishment of an independent and impartial Indian Constitution under Article 315 to Article 323.
Public Service Commission for India.On the basis of • The following provisions assure the Independent
this recommendation, the Union Public Service Functioning of UPSC.
Commission was established in 1926. 1. The conditions of service of the chairman or a
• Statement 2 is correct: In the Government of India member, though determined by the President,
Act 1935, Public Service Commission was cannot be varied to his disadvantage after his
established separately for both the central and the appointment.
state government services. After Independence, 2. The entire expenses including the salaries,
the Union Public Service Commission (UPSC) as allowances, and pensions of the Chairman and
the central recruiting agency was established in members of the UPSC are charged on the
India. It is an independent constitutional body.The Consolidated Fund of India.
provisions regarding the composition of UPSC, 3. The chairman of the UPSC on ceasing to hold office
appointment and removal of its members and the is not eligible for further employment in the
powers and functions of UPSC are provided in Part Government of India or any state.
XIV of the Indian Constitution under Article 315 to 4. A member of the UPSC is eligible for appointment
Article 323. as the Chairman of UPSC or a State Public Service
Commission but not for any other employment in
Q.64) Ans: b the Government of India or any state
Exp: 5. The Chairman or a member of the UPSC can be
removed from office by the President only in the

DPP 2023_DAY 34 32
Download More Test Series & eBooks in Hindi & English from our telegram channel- https://t.me/upsc_success_time1
Contact us :info@onlyias.com

OnlyIAS Nothing Else Visit :dpp.onlyias.in


Contact : +91-7007 931 912

manner and on the grounds mentioned in the candidates for appointment, promotion or transfer
Constitution. Thus, they enjoy the security of in all civil posts are also dealt by UPSC.The
tenure. commission also advises on affairs associated with
the demands and benefits of employees working
Q.66) Ans: c under the All-India Civil Services and injured while
Exp: on duty Also on the payment or expenditure for
• Statement 1 is correct: Article 315 of the any work of an employee of All India Civil Services
Constitution of India states that there shall be a will be borne by the consolidated fund of India.
permanent Union Public Service Commission for
appointment to the various posts of the central Q.68) Ans: b
government services. All the members of the Exp:
commission are appointed by the President of • The Constitution of India makes a provision for the
India with at least half of the members being the establishment of a Joint State Public Service
Civil Servants (working or retired) with no less Commission (JSPSC) for two or more states. While
than ten years of experience in Central or State the UPSC and the SPSC are created directly by the
service. Power of the Union Public Service Constitution, a JSPSC can be created by an act of
Commission is advisory power. It advises the Parliament on the request of the state legislatures
President on matters on all matters related with concerned.
the appointment of the civil services of the • Statement 1 is not correct: The establishment of
governments. The evaluation of the standard and JSPSC in India was first provided by the
efficiencies of the candidates for appointment, Government of India Act, 1935. The Government
promotion or transfer in all civil posts.On all of India Act of 1935 provided for the establishment
matters regarding the discipline and punctuality of of not only a Federal Public Service Commission
the employees of All India Services etc. but also a Provincial Public Service Commission and
• Statement 2 is correct: The Constitution of India Joint Public Service Commission for two or more
has made the Public Service Commission a simple provinces. The 1919 Act provided for the
advisory institution which is required to give advice establishment of a Central Public Service
to the subject sent to it by the President of India or Commission.
by the Governors of the States. But to accept or • Statement 2 is not correct: After the bifurcation of
refuse advice is the absolute discretion of the Punjab into Punjab and Haryana in 1966, a Joint
respective governments. However, the reasons for State Public Service Commission was created for a
non-acceptance of advice of UPSC has to be laid short period for the two states of Punjab and
before each house of the parliament. This Haryana. Since then, no such body has been
provision also ensures the Independent and established.
Impartial functioning of the Commission. • Statement 3 is not correct: A JSPSC presents its
annual performance report to each of the
concerned state governors. Each governor places
Q.67) Ans: c the report before the state legislature.
Exp: • Statement 4 is correct: The Constitution makes a
• Statement 1 is correct: The UPSC is a central provision for the establishment of a Joint State
agency that has great responsibility for conducting Public Service Commission (JSPSC) for two or more
examinations pertaining to Civil Services, states. A JSPSC is not directly created by state
Engineering Services, Defence Services, and legislatures. It can only be created by an act of
Medical Services. Main power of the Union Public Parliament on the request of the state legislatures
Service Commission is its advisory power. It can concerned. Thus, a JSPSC is a statutory and not a
give advice to the President on various matters. constitutional body
This includes matters regarding the discipline and
punctuality of the employees of All India Services. Q.69) Ans: c
On all matters regarding the discipline and Exp:
punctuality of the employees of All India Services. • Statement 1 is not correct: The UPSC consists of a
• Statement 2 is correct: The matters related to the chairman and other members appointed by the
evaluation of the standard and efficiencies of the president of India. The Constitution, without

DPP 2023_DAY 34 33
Download More Test Series & eBooks in Hindi & English from our telegram channel- https://t.me/upsc_success_time1
Contact us :info@onlyias.com

OnlyIAS Nothing Else Visit :dpp.onlyias.in


Contact : +91-7007 931 912

specifying the strength of the Commission has left corporate body or public institution within the
the matter to the discretion of the president, who jurisdiction of the UPSC. Therefore the jurisdiction
determines its composition. of UPSC can be extended by an act made by the
• Statement 2 is correct: No qualifications are Parliament.
prescribed for the Commission’s membership by
the constitution except that one-half of the Q.71) Ans: d
members of the Commission should be such Exp:
persons who have held office for at least ten years About Similipal Biosphere Reserve
either under the Government of India or under the • Similipal Biosphere Reserve is situated in the
government of a state. northern part of Odisha’s Mayurbhanj district.
• Statement 3 is not correct: The chairman and Geographically, it lies in the eastern end of the
members of the Commission hold office for a term eastern ghat. The park derives its name from
of six years or until they attain the age of 65 years, ‘Semul’ which are red silk cotton trees growing in
whichever is earlier. the area. The Park is part of the Mayurbhanj
Elephant Reserve.
• It includes three Similipal Tiger Reserve, Hadagarh
Extra Edge by Only IAS
Wildlife Sanctuary, and Kuldiha Wildlife Sanctuary.
• However, they can relinquish their offices at any
The park has a high biodiversity with about 94
time by addressing their resignation to the
species of orchids and 3,000 species of plants.
president. They can also be removed before the
Among them, Sal is a dominant tree species in the
expiry of their term by the president in the
park.
manner as provided in the Constitution.
• The park is home to the Bengal tiger, Asian
• Usually UPSC consists of 9 to 11 members
elephant, gaur, and chausingha. It is also home to
including chairman.
some beautiful waterfalls like Joranda and
Barehipani Falls. The park was declared a
Q.70) Ans: c biosphere reserve by the Government of India in
Exp: 1994. It has been a part of the UNESCO World
• Statement A is correct : UPSC serves all or any of Network of Biosphere Reserves since 2009.
the needs of a state on the request of the state • The two tribes namely Erenga Kharias and the
governor and with the approval of the president of Mankirdias inhabit the reserve’s forests and
India. It assists the states (if requested by two or practice traditional agricultural activities. Other
more states to do so) in framing and operating dominant tribes include the Ho, Gonda, and
schemes of joint recruitment for any services for Munda among others.
which candidates possessing special qualifications Other Major Protected Areas in Odisha
are required. • Bhitarkanika National Park, Badrama WLS, Chilika
• Statement B is correct : The chairman or a member (Nalaban island) WLS, Hadgarh WLS, Baisipalli
of UPSC is (after having completed his first term) WLS, Kotagarh WLS, Nandankanan WLS, Lakhari
not eligible for reappointment to that office (not Valley WLS, Gahirmatha (Marine) WLS.
eligible for second term).
• Statement C is not correct : The chairman of UPSC Q.72) Ans: c
(on ceasing to hold office) is not eligible for further Exp:
employment in the Government of India or a state About Green bonds
government. • Green bonds are fixed-income financial
o A member of UPSC (on ceasing to hold office) is instruments which are used to fund projects that
eligible for appointment as the chairman of UPSC have positive environmental and or climate
or a State Public Service Commission (SPSC), but benefits. They are just like any regular bond with
not for any other employment in the a key difference that the money raised from
Government of India or a state. investors is used exclusively to finance projects
• Statement D is correct : The additional functions that have a positive environmental impact such as
relating to the services of the Union can be renewable energy and green buildings.
conferred on UPSC by the Parliament. It can also • The popularity of green bonds has been rising
place the personnel system of any authority, considerably, driven primarily by investors

DPP 2023_DAY 34 34
Download More Test Series & eBooks in Hindi & English from our telegram channel- https://t.me/upsc_success_time1
Contact us :info@onlyias.com

OnlyIAS Nothing Else Visit :dpp.onlyias.in


Contact : +91-7007 931 912

embracing socially responsible investing, and not SWIFT is overseen by the G-10 central banks
a better risk and return potential over (Belgium, Canada, France, Germany, Italy, Japan,
conventional bonds. The Netherlands, United Kingdom, United States,
Any organization – such as governments, Switzerland, and Sweden), as well as the European
corporations, and financial institutions – can issue Central Bank, with its lead overseer being the
a green bond. National Bank of Belgium.
Advantage of Green Bonds
• Promotes Clean Energy: Green bonds aim to put Q.74) Ans: c
energy transition and clean energy at the heart of Exp:
India’s economic growth. More specifically, green About Pradhan Mantri Kisan Maan Dhan Yojana
bonds finance projects aimed at energy efficiency, • Pradhan Mantri Kisan Maan Dhan Yojana (PMKVY)
pollution prevention, sustainable agriculture, is a voluntary and contributory pension scheme
fishery and forestry, the protection of aquatic and for the Small & Marginal Farmers (SMFs). It is being
terrestrial ecosystems, clean transportation, clean implemented in order to provide old age
water, and sustainable water management. protection and social security net to the SMFs
• Green bonds are designated bonds intended to by way of pension.
encourage sustainability and to support climate- • Under this scheme, provision has been made for
related or other types of special environmental payment of a minimum fixed pension of Rs. 3,000/-
projects. They also finance the cultivation of to the eligible SMFs, subject to certain exclusion
environmentally friendly technologies and the clauses, on attaining the age of 60 years. The
mitigation of climate change. beneficiary should be a Small and Marginal Farmer.
• The green bond market is globally expanding at a o They must have cultivable land holding up to 2
fast pace and will help India access long-term hectares. The entry age of the scheme is 18 to 40
funds at competitive: years.
o Highlights their green assets/business o The beneficiary should not be SMFs covered
o Positive marketing story under any other statutory social security
o Diversify their investor base schemes such as NPS, Employees’ State
Insurance Corporation scheme, Employees’
Q.73) Ans: d Fund Organization Scheme etc.
Exp: o They should not have opted for Pradhan Mantri
• Statement 1 is correct: SWIFT is a messaging Shram Yogi Maandhan Yojana and National
network used by banks and financial institutions Pension Scheme for Traders and Self-Employed
globally that provides safe and secure exchange of Persons administered by the Ministry of Labour
information pertaining to financial transactions. & Employment.
• Statement 2 is incorrect: It works by assigning each o Should not be from categories of beneficiaries
member institution a unique eight-digit SWIFT ID of higher economic status.
code or a Bank Identification Code that identifies • The eligible beneficiary can opt to become a
not only the bank name but the country, city, and member of the Scheme by subscribing to a Pension
branch. Fund.
• Statement 3 is correct: SWIFT is merely a platform o The beneficiary is required to contribute Rs 100/
that sends messages and does not hold any - per month at median entry age of 29 years.
securities or money. The wide coverage of SWIFT – o The Central Government also contributes to the
covering over 11,000 institutions in more than 200 Pension Fund in equal amounts, managed by the
countries around the world – makes it an almost- Life Insurance Corporation, which is also
universally accepted system. It counts central responsible for pension pay out. As the Scheme
banks of countries like the US, UK, Germany, has an entry age of 18 to 40 years, no beneficiary
France, Japan, India, China, Singapore and others has yet attained the age of 60 to be eligible for
among its list of overseers. payment.
• SWIFT claims to be neutral. Its shareholders,
consisting of 3,500 firms across the globe, elect the Q.75) Ans: b
25-member board, which is responsible for Exp:
oversight and management of the company.

DPP 2023_DAY 34 35
Download More Test Series & eBooks in Hindi & English from our telegram channel- https://t.me/upsc_success_time1
Contact us :info@onlyias.com

OnlyIAS Nothing Else Visit :dpp.onlyias.in


Contact : +91-7007 931 912

• Statement 1 is incorrect: Pradhan Mantri urja • The National Urban Digital Mission aims to
Ganga project is a gas pipeline project that aims to institutionalise a citizen-centric and ecosystem-
provide cooking gas to people. It was launched in driven approach to urban governance and service
2016 in Varanasi and later extended to people in delivery. The NUDM will be completed in 2022
the states of Bihar, West Bengal, Odisha and cities by 2022. Further, it will be expanded across
Jharkhand. The government is taking the initiative all cities and towns in India by 2024.
to connect the eastern states with the national gas • The digital infrastructure will help cities to
grid. consolidate and cross-leverage the various digital
o The total length of the pipeline under the Pradhan initiatives of the Ministry of Housing and Urban
Mantri Urja Ganga Project is approximately 3,384 Affairs. It will also enable the cities and towns to
km, out of which 766 km of pipeline is in Odisha benefit from holistic and diverse forms of support.
state and the balance 2,618 km is in the states of Such as satisfying the needs and local challenges of
Uttar Pradesh, Bihar, Jharkhand, West Bengal & the towns and cities.
Assam. • National Urban Digital Mission (NUDM) & Several
o The project will not just supply CNG to automobiles Digital Initiatives Launched For Transforming
and cooking gas to household kitchens in cities Urban Governance
along the route, but also to industries to meet their • Shri Hardeep S Puri, MoS, I/C, Housing and Urban
feedstock or fuel requirements. The project is Affairshas stated that National Urban Digital Missi
being implemented by GAIL.
• Statement 2 is correct: Line of Credit is a credit Q.77) Ans: b
facility extended by a bank or any other financial Exp:
institution to a government, business or individual About Authorised Economic Operator
customer. The borrower can access funds from the • Authorised Economic Operator (AEO) is a
LOC at any time as long as they do not exceed the programme (2007) under the aegis of the World
maximum amount (or credit limit) set in the Customs Organization (WCO) SAFE Framework of
agreement and meet any other requirements such Standards to secure and facilitate global trade.
as making timely minimum payments. • It aims to enhance international supply chain
o It is not a grant but a ‘soft loan’ provided on security and facilitate the movement of goods.
concessional interest rates to developing • AEO is a voluntary compliance programme.
countries, which has to be repaid by the borrowing • Under this programme, an entity engaged in
government. international trade is approved by Customs as
o The projects under LOCs spread over different compliant with supply chain security standards and
sectors like agriculture, infrastructure, telecom, granted AEO status & certain benefits.
railway, transmission, power, renewable energy • Benefits of AEO status include expedited clearance
etc. These help to promote exports of Indian goods times, fewer examinations, improved security and
and services, as 75% of the value of the contract communication between supply chain partners.
must be sourced from India. • Recent context: The Central Board of Indirect Taxes
& Customs (CBIC) has inaugurated the online filing
Q.76) Ans: c of Authorised Economic Operators (AEO)
Exp: applications.
• National Urban Digital Mission: • Extra Edge by OnlyIAS:
• The National Urban Digital Mission was launched o About SAFE Framework
to build the shared digital infrastructure that will o In June 2005 the WCO Council adopted the
strengthen the capacity of the urban ecosystem to Framework of Standards to Secure and Facilitate
solve complex problems. It has been launched by Global Trade (SAFE Framework) that would act as
the Ministry of Housing and Urban affairs (MoHUA) a deterrent to international terrorism, to secure
in partnership with the Ministry of Electronics and revenue collections and to promote trade
Information Technology (MEITY). facilitation worldwide.
• The digital infrastructure will be built across three o It prescribes baseline standards that have been
pillars of People, Process and platform. The tested and are working well around the globe.
infrastructure will provide holistic support to cities
and towns. Q.78) Ans: b

DPP 2023_DAY 34 36
Download More Test Series & eBooks in Hindi & English from our telegram channel- https://t.me/upsc_success_time1
Contact us :info@onlyias.com

OnlyIAS Nothing Else Visit :dpp.onlyias.in


Contact : +91-7007 931 912

Exp: Q.80) Ans: b


• The Judges Enquiry Act (1968) regulates the Exp:
procedure relating to the removal of a judge of the About Param Pravega
High Court by the process of impeachment. • Param pravega designed by the Centre for
1. A removal motion signed by 100 members (in the Development of Advanced Computing (C-DAC). It
case of Lok Sabha) or 50 members (in the case of is the largest supercomputer in an Indian academic
Rajya Sabha) is to be given by the institution. It was installed and commissioned
Speaker/chairman. under the National Supercomputing Mission
2. The Speaker/Chairman may admit the motion or (NSM).
refuse to admit it. • The system is expected to power diverse research
3. If it is admitted, then the Speaker/Chairman is to and educational pursuits. It has a supercomputing
constitute a three-member committee to capacity of 3.3 petaflops (1 petaflop equals a
investigate the charges. quadrillion, or 1015 operations per second).
4. If the committee finds the judge to be guilty of
misbehaviour or suffering from an incapacity, the
National Supercomputing Mission (NSM)
house can take up the consideration of the motion.
The mission was announced in 2015. NSM
5. After the motion is passed by each house of
envisaged setting up a network of 70 high-
Parliament by special majority, an address is
performance computing facilities with an aim to
presented to the President for removal of the
connect national
judge.
academic and R&D institutions across India over a
6. Finally, the President passes an order removing the
seven-year period at an estimated cost of Rs 4500
judge.
Crores. Parent body: Ministry of Electronics and IT
• The grounds of removal of a judge of the High
(MeitY) and Department of Science and Technology
Court are two: proved misbehaviour or incapacity.
(DST).
• The procedure of impeachment of a judge of the
Nodal Agencies of NSM: Centre for Development of
High Court is similar to that of the Supreme Court.
Advanced Computing (C-DAC), Pune, and the Indian
• No judge of a High Court has been impeached so
Institute of Science (IISc), Bengaluru.
far.
Under NSM, the long-term plan is to build a strong
base of 20,000 skilled persons over the next five
Q.79) Ans: b
years who will be equipped to handle the
Exp:
complexities of supercomputers.
About surety bond
In 2020, a RTI reply revealed that India has
• A surety bond is a legally binding contract entered
produced just three supercomputers since 2015
into by three parties—the principal, the obligee
under NSM:
and the surety. The obligee, usually a government
• PARAM Shivay installed in IIT-BHU, Varanasi
entity, requires the principal, typically a business
with 837 TeraFlop capacity
owner or contractor, to obtain a surety bond as a
• PARAM Shakti at IIT-Kharagpur with 1.66
guarantee against future work performance.
PetaFlop capacity
• Surety bonds are mainly aimed at infrastructure
• PARAM Brahma at ISER-Pune, has a capacity of
development, mainly to reduce indirect cost for
797 TeraFlop.
suppliers and work- contractors thereby
diversifying their options and acting as .
a substitute for bank guarantee. Q.81) Ans: b
• Surety bond is provided by the insurance company Exp:
on behalf of the contractor to the entity which is • Statement 1 is incorrect: A neutrino is a subatomic
awarding the project. Surety bonds protect the particle with no electric charge. They are
beneficiary against acts or events that impair the considered to be the second most abundant
underlying obligations of the principal. particle in the universe, after photons, or light
They guarantee the performance of a variety of particles. Predicted in 1931, neutrinos were
obligations, from construction or service contracts detected for the first time in 1959 by Clyde Cowan
to licensing and commercial undertakings. and Fred Reines. They come in three ‘flavours’ or

DPP 2023_DAY 34 37
Download More Test Series & eBooks in Hindi & English from our telegram channel- https://t.me/upsc_success_time1
Contact us :info@onlyias.com

OnlyIAS Nothing Else Visit :dpp.onlyias.in


Contact : +91-7007 931 912

‘types’, and each flavour is associated with a light • Statement 1 is correct: The Indus Waters Treaty,
elementary particle. They are, 1960 is a water distribution treaty between India
o Electron-neutrino is associated with the electron; and Pakistan. It was brokered by the World Bank.
o Muon-neutrino with the muon and The Indus system comprises Indus, Jhelum,
o Tau-neutrino with the tau particle. Chenab, Ravi, Beas, and Sutlej. The basin is mainly
• Statement 2 is correct: They are not easy to catch, shared by India and Pakistan, with a small share
as they do not carry a charge, as a result of which for China and Afghanistan.
they do not interact with matter. They also might • Statement 2 is incorrect: Under the Indus treaty,
have unique properties that would help explain all the waters of three eastern rivers namely Ravi,
why the universe is made of matter instead of Sutlej, and Beas were allocated to India for
antimatter. exclusive use. On the other hand, the waters of
• Subatomic particles that make up antimatter have Western rivers namely Indus, Jhelum, and Chenab
properties that are opposite to the subatomic were allocated to Pakistan.
particles of normal matter. o But India can use the water in western rivers for
• Protons, neutrons and electrons (subatomic non-consumptive needs like irrigation, storage,
particles of normal matter) are among the 12 and even
quarks and leptons have been discovered so far. o for electricity production. Thus, India has been
given the right to generate hydroelectricity
Q.82) Ans: b through a run of the river projects on the
Exp: western rivers, subject to specific criteria for
• Recently, A US patient became the First woman design and operation.
reported to be cured of HIV after stem
cell transplant (SCT). Stem cells are special human
Permanent Indus Commission
cells that are able to develop into many different
The Permanent Indus Commission is a bilateral
cell types. Stem cells provide new cells for the body
commission of officials from India and Pakistan,
as it grows and replaces specialized cells that are
created to
damaged or lost.
implement and manage goals of the Indus Waters
• They have two unique properties that enable
Treaty, 1960.
them to do this:
The Commission according to the treaty must meet
o They can divide over and over again to produce
regularly at least once a year, alternately in India and
new cells. As they divide, they can change into the
Pakistan.
other types of cells that make up the body.
The functions of the Commission include:
• Stem Cell Classification: Based on the cell
• To study and report to the two Governments on
type/tissue of origin, stem cells are classified as
any problem relating to the development of the
‘Somatic Stem Cells’ (SSCs), and ‘Embryonic Stem
waters of the rivers.
Cells’ (ESCs) (refer infographic
• To solve disputes arising over water sharing.
• Embryonic stem cells. The embryonic stem cells
• To arrange technical visits to projects’ sites and
used in research today come from unused
critical river head works.
embryos. They are donated to science. These
• To undertake, once in every five years, a
embryonic stem cells are pluripotent. This means
general tour of inspection of the Rivers for
that they can turn into more than one type of cell.
ascertaining the facts.
• Adult stem cells. There are 2 types of adult stem
cells. One type comes from fully developed tissues
such as the brain, skin, and bone marrow. They are Q.84) Ans: a
more likely to generate only certain types of cells. Exp:
For example, a stem cell that comes from the liver Ayushman Bharat Digital Mission
will only make more liver cells. Hence option (b) is • The National Health Authority has announced the
correct. integration of its flagship Ayushman Bharat Digital
Mission with the Aarogya Setu app, allowing users
Q.83) Ans: a to create the 14-digit unique Ayushman Bharat
Exp: Health Account numbers from the app.About The
Ayushman Bharat Digital Mission will provide a

DPP 2023_DAY 34 38
Download More Test Series & eBooks in Hindi & English from our telegram channel- https://t.me/upsc_success_time1
Contact us :info@onlyias.com

OnlyIAS Nothing Else Visit :dpp.onlyias.in


Contact : +91-7007 931 912

digital health ID to the people who will hold their revolutionaries like Chandrasekhar Azad and
health records. Bhagat Singh.
• It is a digital health ecosystem under which every • He along with Rashbehari Bose attacked Viceroy
Indian citizen will now have unique health IDs, Hardinge and injured him while he was entering
digitised health records with identifiers for the new capital of Delhi. Sanyal was closely
doctors and health facilities. The scheme will come involved in the plans for the Ghadar Conspiracy. He
under the Ayushman Bharat Pradhan Mantri Jan was sentenced to life for his involvement in the
Arogya Yojana. conspiracy and was imprisoned at Cellular Jail.
• It comprises six key building blocks — HealthID, He was also jailed by the British for his involvement
DigiDoctor, Health Facility Registry, Personal in the Kakori Conspiracy. Sachindra Nath Sanyal
Health Records, e-Pharmacy and Telemedicine. died in Gorakhpur Jail on 7th February 1942.
The National Health Authority has been given the
mandate to design, build, roll-out and implement Q.86) Ans: d
the mission in the country. Exp:
• The core building blocks of the mission is that the • Statement 1 is correct: The Supreme Court
health ID, DigiDoctor and Health Facility Registry enquiries into the conduct and behavior of the
shall be owned, operated and maintained by the chairman and members of the Union Public
Government of India. Private stakeholders will Service Commission on a reference made by the
have an equal opportunity to integrate and create President. If it finds them guilty of misbehavior, it
their own products for the market. The core can recommend to the President for their removal.
activities and verifications, however, remain with The advice tendered by the Supreme Court in this
the government. Under the Mission, every Indian regard is binding on the President.
will get a Health ID card that will store all medical • Statement 2 is correct: The Supreme Court is
details of the person including prescriptions, authorized to withdraw the cases pending before
treatment, diagnostic reports and discharge the High Courts and dispose of them by itself. It can
summaries. also transfer a case or appeal pending before one
• Health ID is a randomly generated 14 digit number High Court to another High Court.
used for the purposes of uniquely identifying • Statement 3 is correct: The Supreme Court is a
persons, authenticating them, and threading their self-correcting agency, it has the power to review
health records (only with their informed consent) its own judgment or order and is not bound by its
across multiple systems and stakeholders. The previous decision and can depart from it in the
citizens will be able to give their doctors and interest of justice or community welfare.
health providers one-time access to this data
during visits to the hospital for consultation. Q.87) Ans: c
• The mission aims to liberate citizens from the Exp:
challenges of finding the right doctors, seeking About Nai Roshni Scheme
appointment, payment of consultation fee, • Nai Roshni Scheme is the Central Sector Scheme
making several rounds of hospitals for started in 2012-13 and comes under the Ministry
prescription sheets, among several others and will of Minority Affairs. It was launched For women
empower people to make an informed decision to belonging to minority communities in the age
avail the best possible healthcare. Hence option group of 18 to 65 years.
(a) is correct • The objective of the Nai Roshni scheme is to
empower and instil confidence among minority
Q.85) Ans: a women, including their neighbours from other
Exp: communities living in the same village/locality, by
About Sachindra Nath Sanyan providing knowledge, tools and techniques for
• Sachindra Nath Sanyan was the founder of the interacting with Government systems, banks and
Hindustan Republican Association also known as other institutions at all levels. It is implemented
Hindustan Socialist Republican Association. It was with the help of NGOs, Civil societies and
created to carry out armed resistance against the Government Institutions all over the country.
British Empire in India. He was a mentor for • It includes various training modules like
Leadership of women, Educational Programmes,

DPP 2023_DAY 34 39
Download More Test Series & eBooks in Hindi & English from our telegram channel- https://t.me/upsc_success_time1
Contact us :info@onlyias.com

OnlyIAS Nothing Else Visit :dpp.onlyias.in


Contact : +91-7007 931 912

Health and Hygiene, Swachch Bharat, Financial • The Administrative Reforms Commission headed
Literacy, Life Skills, Legal Rights of Women, Digital by Late Morarji Desai in 1966 recommended the
Literacy and Advocacy for Social and behavioral setting up of the institution of Lokayukta.
change. • The Lokpal and Lokayukta Act, 2013, commonly
About National Overseas Scholarship Scheme: known The Lokpal Act was passed by the
• The National Overseas Scholarship Scheme, Parliament of India in December 2013. The law
the Central Sector Scheme, was launched in 2008. gave states the autonomy to frame their own laws.
Its nodal ministry is the Ministry of Education.
• To award scholarships to meritorious students of Q.89) Ans: d
economically weaker sections to deter them from Exp:
dropping out at class VIII and encourage them to About Fundamental Duties
continue their education at the secondary stage. • The Fundamental Duties were incorporated in Part
• The scheme envisages the award of one lakh fresh IV-A of the Constitution by the Constitution 42nd
scholarships every year to selected students of Amendment Act, 1976, during Emergency under
classIX and their continuation/renewal in classes X Indira Gandhi’s government. The amendment
to XII for study in a State Government, came at a time when elections stood suspended
Government-aided and Local body schools. An and civil liberties curbed.
amount of Rs. 12000 (Rs.1000/- per month) per • Today, there are 11 Fundamental Duties described
student per annum. under Article 51-A, of which 10 were introduced by
• Students are selected for award of scholarships the 42nd Amendment and the 11th was added by
through an examination conducted by the State/ the 86th Amendment in 2002, during Atal Bihari.
UT Governments. Scholarships are disbursed • These are statutory duties, not enforceable by
directly into the bank accounts of students by law, but a court may take them into account while
electronic transfer through the Public Financial adjudicating on a matter. The idea behind their
Management System (PFMS) following DBT mode. incorporation was to emphasise the obligation of
the citizen in exchange for the Fundamental Rights
Q.88) Ans: a that he or she enjoys.
Exp: • The concept of Fundamental Duties is taken from
• Statement 1 is correct: Lokayukta is an anti- the Constitution of Russia.
corruption authority or ombudsman – an official The 11 Fundamental Duties are -
appointed by the government to represent the • To abide by the constitution and respect its ideals
interests of the public. Most importantly, it and institutions, the National Flag and the National
investigates allegations of corruption and mal- Anthem.
administration against public servants and is • To cherish and follow the noble ideals which
tasked with speedy redressal of public grievances. inspired our national struggle for freedom
• Statement 2 is correct: Lokayuktas are the state To uphold and protect the sovereignty, unity and
equivalents of the central Lokpal. The Lokayukta is integrity of India — it is one of the preeminent
usually a former High Court Chief Justice or former national obligations of all the citizens of India.
Supreme Court judge and has a fixed tenure. The • To defend the country and render national service
Chief Minister selects a person as the Lokayukta when called upon to do so.
after consultation with the High Court Chief • To promote harmony and the spirit of common
Justice, the Speaker of the Legislative Assembly, brotherhood amongst all the people of India
the Chairman of the Legislative Council, Leader of transcending religious, linguistic and regional or
Opposition in the Legislative Assembly and the sectional diversities; to renounce practices
Leader of Opposition in the Legislative Council. The derogatory to the dignity of women.
appointment is then made by the Governor. • To value and preserve the rich heritage of our
• Statement 3 is incorrect: Once appointed, composite culture — our cultural heritage is one
Lokayukta cannot be dismissed nor transferred by of the noblest and richest, it is also part of the
the government, and can only be removed by heritage of the Earth.
passing an impeachment motion by the state • To protect and improve the natural environment
assembly. including forests, lakes, rivers and wild life and to
have compassion for living creatures.

DPP 2023_DAY 34 40
Download More Test Series & eBooks in Hindi & English from our telegram channel- https://t.me/upsc_success_time1
Contact us :info@onlyias.com

OnlyIAS Nothing Else Visit :dpp.onlyias.in


Contact : +91-7007 931 912

• To develop the scientific temper, humanism and Q.91) Ans: c


the spirit of inquiry and reform. Exp:
• To safeguard public property and to abjure Synthetic Biology
violence. • Synthetic biology refers to the science of using
• To strive towards excellence in all spheres of genetic sequencing, editing, and modification to
individual and collective activity so that the nation create unnatural organisms or organic molecules
constantly rises to higher levels of endeavour and that can function in living systems. Synthetic
achievement. biology enables scientists to design and synthesise
• Who is a parent or guardian to provide new sequences of DNA from scratch.
opportunities for education to his child or, as the o The term ‘synthetic biology’ was first used by
case may be, ward between the age of six and Barbara Hobomin in 1980, to describe bacteria
fourteen years. Hence option ( d) is correct. that had been genetically engineered using
recombinant DNA technology.
Q.90) Ans: c Applications of Synthetic biology
Exp: • It has applications in various fields. For instance, it
Prime Minister’s Development Initiative for North can help in a) Developing synthetic organisms for
East vaccination, b) Creating natural products in a lab
• Union Budget 2022-23 provided for a new scheme, such as vanillin, extracted from vanilla seeds, can
Prime Minister’s Development Initiative for North be grown in yeasts with additional plant genomes.
East (PM-DevINE) , which will be implemented • Pharmaceutical industry: Synthetic biology can be
through the North-Eastern Council. PM-DevINE used to make natural compounds such as
will fund infrastructure, in the spirit of PM artemisinin used for the treatment of malaria and
GatiShakti, and social development projects based Car T cell therapy for cancer treatment.
on felt needs of the North-East states. • In the fashion industry: Some companies are
• PM-DevINE scheme is being implemented with the exploring the possibility of dyeing jeans without
following objectives- producing hazardous waste using synthetic
o To enable livelihood activities for youth and biology.
women. • In Agriculture: Companies using synthetic biology
o To fill gaps in various sectors. to deliver fixed nitrogen to plants instead of using
• The PM-DevINE scheme will be implemented fertilisers, engineering microbes to create food
through the North-Eastern Council.An initial additives or brew proteins.
allocation of Rs. 1,500 crores will be made for the
PM-DevINE scheme. Q.92) Ans: c
• Under the PM-DevINE scheme, projects can be Exp:
recommended by Centre and North East States as About the Hoyasala architecture
well but priority will be given to projects posed by • Hoyasala architecture is a building style that
the states. Some of the projects to be evolved between the 11th to 14th centuries under
implemented are: the Hoyasala Empire’s rule in the southern Deccan
o Dedicated Services for the Management of region and Kaveri River Valley. They are easily
Paediatric and Adult Hematolymphoid Cancers in distinguishable from other medieval temples by
North East India, Guwahati their highly original star-like ground-plans and a
o Construction of Aizawl bypass on western side, profusion of decorative carvings.
gap funding for passenger ropeway system for • Hoyasala rulers were influenced by the western
Pelling to Sanga-Choeling in West Sikkim Chalukyan architecture.
o Gap funding for eco-friendly Ropeway (Cable • Hoyasala temples have several shrines arranged
Car) from Dapper to Bhaleydhunga in South around a central pillared hall in the form of an
Sikkim intricately designed star (stellate-plan).
o Pilot project for the construction of Bamboo o The garbha griha (sanctum sanctorum) houses a
Link Road at different locations in various centrally placed murti (enshrined icon) on a pitha
districts in Mizoram the scheme will not be a (pedestal).
substitute for existing central and state schemes. o The temple craftsmen carved their sculptures
intricately because they were made of soapstone,

DPP 2023_DAY 34 41
Download More Test Series & eBooks in Hindi & English from our telegram channel- https://t.me/upsc_success_time1
Contact us :info@onlyias.com

OnlyIAS Nothing Else Visit :dpp.onlyias.in


Contact : +91-7007 931 912

a relatively soft stone. These sculptures can be Q.94) Ans: a


seen in the gods’ jewelry that adorns the temple Exp:
walls. Election Commission of India
o Both open and closed mandapas can be found in • Election Commission is a permanent and an
Hoysala architecture. independent body established by the Constitution
o Hoysala temples have circular pillars in their under article 324. The power of superintendence,
mandapas. Each pillar has four sculpted figures on direction and control of elections to: Parliament,
the top brackets. Etc. state legislatures, the office of president of India
• The Union Ministry of Culture has announced that and the office of vice-president of India shall be
the Hoysala temples of Belur, Halebid and vested in the election commission.
Somnathapura in Karnataka have been selected as • The Election Commission shall consist of the chief
India’s nomination for UNESCO’s list of World election commissioner and such number of other
Heritage sites for the year 2022-2023. The temples election commissioners as the president may
representing the Hoyasala Architecture style are decide. The appointment of the chief election
called ‘The Sacred Ensembles of Hoyasala’. commissioner and election commissioners shall be
• They have been on the tentative list of UNESCO made by the president.
since 2014 and represent one of the highest points • From 1950 till 15 October 1989, the election
of human creative genius and stand testimony to commission functioned as a single member body:
the rich historical and cultural heritage of the consisting of the Chief Election Commissioner. On
country. They are protected monuments of the 16 October 1989, the president appointed two
Archaeological Survey of India (ASI), which takes more election commissioners. But the two posts of
care of their conservation and maintenance. election commissioners were abolished in January
1990. Hence option (a) is incorrect.
Q.93) Ans: a • Again, in October 1993, the president appointed
Exp: two more election commissioners. Since then, the
• Statement 1 is correct: Poliomyelitis (polio) is a Election Commission has been functioning as a
highly infectious viral disease that largely affects multi-member body consisting of three election
children under 5 years of age. commissioners.
o It may affect the spinal cord causing muscle • Statement 2 is correct: The Election commission
weakness and paralysis. decides the election schedule for the conduct of
o Transmitted by person-to-person spread mainly both general elections and bye-elections. Election
through the faecal-oral route or, less frequently, Commission resolves the disputes relating to
by contaminated water or food). splits/mergers of recognized political parties. Its
o Wild poliovirus (WPV) is the most commonly decision can be challenged in the court of law.
known form of the poliovirus.
o There are 3 strains of WPV- type 1, type 2 and Q.95) Ans: a
type 3. type 2 was eradicated in 1999 no case of Exp:
type 3 has been found since 2012 .As of 2020, • The constitution of India provided a parliament for
type 1 affects two countries: Pakistan and India as the legislative organ. The article 79-122
Afghanistan. deals with provisions for parliament which includes
• Statement 2 is incorrect: Lassa fever is a zoonotic the provisions regarding eligibility criteria,
disease caused by the Lassa virus.It was first functions, rules of disqualification etc.
discovered in 1969 in Nigeria. It is endemic in parts • Statement 1 is correct: Article 102 of the
of West Africa including Sierra Leone, Liberia, Constitution provides for disqualification
Guinea and Nigeria. Symptoms include bleeding, conditions for a member of either House of
Difficulty breathing; vomiting; facial swelling; pain Parliament. The member shall be disqualified if: He
in the chest, etc. holds an office of profit (under the Government of
• It spreads through Infected rats, rarely spreading India or the Government of any State) other than
via direct contact with a sick person’s body fluids. the office mentioned in the law of parliament.
Ribavirin an antiviral drug that is used for its o If he is of unsound mind and so declared by
treatment and maintaining hydration, oxygen competent court;
levels etc o If he found to be an undischarged insolvent;

DPP 2023_DAY 34 42
Download More Test Series & eBooks in Hindi & English from our telegram channel- https://t.me/upsc_success_time1
Contact us :info@onlyias.com

OnlyIAS Nothing Else Visit :dpp.onlyias.in


Contact : +91-7007 931 912

o If he is not a citizen of India, or has voluntarily o He can take all other necessary steps including
acquired the citizenship of a foreign State, or is the suspension of the constitutional provisions
under any acknowledgment of allegiance or relating to anybody or authority in the state.
adherence to a foreign State; • Statement 1 is correct: when the president's rule
o If he is disqualified under any law made by is imposed in a State,the president dismisses the
Parliament. state council of ministers headed by the chief
• Statement 2 is incorrect: Other Grounds for minister. The state governor, on behalf of the
Disqualification If a person is convicted for corrupt President, carries on the state administration with
electoral practices. the help of the chief secretary of the state or the
o If the person gets convicted for certain offences – advisors appointed by the president. This is the
like: IPC; Civil Rights Act, 1955, UAPA Act, 1967 reason why a proclamation under Article 356 is
etc. popularly known as the imposition of 'president
o If the person gets convicted of any offence where rule' in a State.
imprisonment is not less than 2 years. • Statement 2 is correct: The President either
suspended or dissolved the state legislative
Q.96) Ans: a assembly, The Parliament passes the state
Exp: legislative bills and the state budget.
• In 1976, the fundamental duties of citizens were • Statement 3 is incorrect: The President can not
added in the Constitution on the assume to himself the powers vested in the
recommendations of the Swaran Singh concerned state high court or suspend the
Committee. Article 51A of Part IV A of the Indian provisions of the constitution relating to it. In
Constitution deals with Fundamental Duties. other words, the constitutional position, status,
• Statement 1 is correct: Under article 51 A (h), it is powers and functions of the concerned state high
a duty to develop the scientific temper, humanism, court remain the same even during the president's
and spirit of inquiry and reform. Rule.
• Statement 2 is incorrect: Under article 51 A (a),
duty to abide by the Constitution and respect its Q.98) Ans: c
ideals and institutions, the National Flag and the Exp:
National Anthem. There is nowhere mentioned in • Statement 1 is incorrect: As a court of record, a
the Constitution to sing the National Anthem high court also has the power to review and correct
during morning assembly in the school. its own judgement or order or decision, even
• Statement 3 is correct: Under article 51 A (f) it is though no specific power of review is conferred on
the duty of a citizen to value and preserve the rich it by the Constitution. The supreme court on the
heritage of our composite culture. other hand, has been specifically conferred with
• The Indian Constitution was amended by the 42nd the power of review by the constitution.
Amendment Act of 1976, which introduced ten • Statement 2 is correct: A court of record is a court
Fundamental Duties. The 11th Fundamental Duty whose acts and proceeding are enrolled for
was later added to the list by the 86th Amendment perpetual memory and testimony. These records
Act of 2002. are used with high authority and their truth
cannot be questioned. These records are admitted
Q.97) Ans: a to be evidentiary value and cannot be questioned
Exp: when produced before any subordinate Courts.
Consequences of President's Rule They are recognised as legal precedents and legal
• The president acquire the following extraordinary references.
powers when the president's rule imposed in a • Statement 3 is correct: Article 29: The supreme
State: court shall be a court of record and shall have the
o He can take up the function of the state power of such a court including the power to
government and powers vested in the punish for contempt of itself. Article 215: every
government or any other executive authority in high court shall be a court record and shall have all
the state. the powers of such a court including the power to
o He can declare that the power of the legislature punish for contempt of itself.
is to be exercised by the parliament.

DPP 2023_DAY 34 43
Download More Test Series & eBooks in Hindi & English from our telegram channel- https://t.me/upsc_success_time1
Contact us :info@onlyias.com

OnlyIAS Nothing Else Visit :dpp.onlyias.in


Contact : +91-7007 931 912

Q.99) Ans: d o INS Khanderi was the second submarine. It was


Exp: commissioned in 2019.
• Statement 1 is correct: Kerala has deposits of o INS Karanj was the third submarine. It was
monazite and thorium, bauxite clay. Goa has iron commissioned in March 2021.
ore deposits. The North-Western Region belt o INS Vela was the fourth submarine. It was
extends along Aravali in Rajasthan and part of commissioned in November 2021.
Gujarat and minerals are associated with the o INS Vagir is the fifth submarine.
Dharwar system of rocks. Copper, zinc have been o INS Vasgheer is the sixth submarine.
major minerals. • The engines of the submarine are powered by four
• Statement 2 is correct: Rajasthan is rich in diesel engines and 360 battery cells. The weapons
building stones i.e. sandstone, granite, marble. of the submarine are integrated with its SUBTICS
Gypsum and Fuller’s earth deposits are also (Submarine Tactical Integrated Combat System).
extensive. Dolomite and limestone provide raw The submarine has a sonar system that is capable
materials for the cement industry. Gujarat is of LOFAR (Low Frequency Analysis and Ranging).
known for its petroleum deposits. You may be
knowing that Gujarat and Rajasthan both have rich
sources of salt.
• Statement 3 is correct: The Himalayan beltis
another mineral belt where copper, lead, zinc,
cobalt and tungsten are known to occur. They
occur in both the eastern and western parts.
Assam valley has mineral oil deposits. Oil
resources are also found in off-shore-areas near
Mumbai Coast (Mumbai High).

Extra Edge by Only IAS


Distribution of minerals in India:
The South-Western Plateau Region. This belt
extends over Karnataka, Goa and contiguous Tamil
Nadu uplands and Kerala.
This belt is rich in ferrous metals and bauxite. It also
contains high grade iron ore, manganese and
limestone. This belt lacks coal deposits except
Neyveli lignite. This belt does not have as diversified
mineral deposits as the north-eastern belt.

Q.100) Ans: c
Exp:
About Project-75
• Project-75 envisages indigenous construction of
submarines equipped with the state-of-the-art Air
Independent Propulsion system at an estimated
cost of Rs. 43,000 crore.
• Under this project, the Indian Navy intends to
acquire Six Diesel-Electric Submarines, which will
also feature Advanced Air-Independent Propulsion
systems to enable them to stay submerged for
longer duration and substantially increase their
operational range. Six submarines are:
o INS Kalvari was the first submarine delivered
under Project -75. It was delivered in 2015.

DPP 2023_DAY 34 44
Download More Test Series & eBooks in Hindi & English from our telegram channel- https://t.me/upsc_success_time1
Download more eBooks & Test Series in Hindi and English
from our
Join Telegram- Telegram Channel-
https://t.me/upsc_success_time1
https://t.me/upsc_success_time1

Join Telegram Channel - Click Here

You might also like